При параллельном соединении конденсаторов: Схемы соединения конденсаторов — расчет емкости

Содержание

При последовательном соединении конденсаторов их суммарная емкость — советы электрика

Комбинации конденсаторов: последовательные и параллельные

Физика > Комбинации конденсаторов: последовательные и параллельные

Изучите соединения конденсаторов – последовательные и параллельные. Как выглядят последовательно и параллельно соединенные конденсаторы, емкость, схемы.

Конденсаторы можно использовать в последовательных или параллельных цепях.

Задача обучения

  • Вывести общую емкость конденсаторов в разных типах соединения.

Основные пункты

 – последовательное соединение.

  • Собщая = С1 + С2 + … + Сn – параллельное соединение.
  • Если конденсаторы пребывают в последовательном или параллельном подключении, то лучше упростить схему и решить по частям.

Термины

  • Конденсатор – электронная составляющая, способная сберегать электрический заряд.
  • Схема – маршрут электрического тока, представленный отдельными электронными составляющими, вроде резисторов, транзисторов, конденсаторов, катушек индуктивности и т.д.
  • Конденсаторы можно применить в различных комбинациях цепей. Они могут быть последовательными (несколько конденсаторов расположены на одном пути) и параллельными (на разных путях).

Последовательные конденсаторы

Посмотрим на схему, где конденсаторы соединены последовательно. Обратная общая емкость достигает суммы обратных значений емкости каждого отдельного конденсатора:

Здесь показаны последовательные конденсаторы С1, С2, С3 и до Сn

Последовательно соединенные конденсаторы можно также выразить:

Параллельные конденсаторы

Суммарная емкость в параллельном соединении конденсаторов находится при обычном добавлении отдельных емкостей каждого конденсатора.

Здесь показаны параллельные конденсаторы С1, С2, С3 и до Сn

Собщая = С1 + С2 + … + Сn.

Последовательные и параллельные конденсаторы

Бывает так, что схема вмещает сразу два типа соединения (параллельное и последовательное соединение конденсаторов). Чтобы отыскать общую емкость, нужно разделить цепочку на отдельные сегменты.

Задачу можно сделать проще, если сначала решить проблему с последовательным соединением, а потом заняться параллельным

В (а) расположены последовательные конденсаторы, выступающие параллельными C3. Если найти емкость для последовательных, то можно потом заняться вычислением для одного конденсатора. Оно будет равняться примерно 0.83 мкФ.

Обратите внимание

В одновременном выводе двух оставшихся конденсаторов, можно приплюсовать их емкости и получим общую – 8.83 мкФ.

(1

Источник: https://v-kosmose.com/fizika/kombinatsii-kondensatorov-posledovatelnyie-i-parallelnyie/

Последовательное и параллельное соединение конденсаторов

На практике часто используются тела, обладающие малыми (и очень малыми) размерами, которые могут накопить большой заряд, при этом имея небольшой потенциал. Такие объекты называют конденсаторами. Одна из основных характеристик конденсатора – это его емкость.

Имея в резерве набор конденсаторов, обладающих разными параметрами, можно расширить спектр величин емкостей и диапазон рабочих напряжений, если применять их соединения.

Различают три типа соединений конденсаторов: последовательное, параллельное и смешанное (параллельное и последовательное).

Последовательное соединение конденсаторов

Последовательное соединение изконденсаторов изображено на рис. 1

Здесь (рис.1) положительная обкладка одного конденсатора соединяется с отрицательной обкладкой следующего конденсатора. При таком соединении обкладки соседних конденсаторов создают единый проводник. У всех конденсаторов, соединенных последовательно на обкладках имеются равные по величине заряды. Электрическая емкость последовательного соединения конденсаторов вычисляется по формуле:

где– электрическая емкость i-го конденсатора.

Если емкости конденсаторов при последовательном соединении равны, то емкость последовательного их соединения составляет:

где N – количество последовательно соединенных конденсаторов. При этом предельное напряжение (U), которое выдержит подобная батарея конденсаторов составит:

где– предельное напряжение каждого конденсатора соединения. При последовательном соединении конденсаторов следует следить за тем, чтобы ни на один из конденсаторов батареи не падало напряжение, превышающее его максимальное рабочее напряжение.

Параллельное соединение конденсаторов

Параллельное соединение N конденсаторов изображено на рис. 2.

При параллельном соединении конденсаторов соединяют обкладки, обладающие зарядами одного знака (плюс с плюсом; минус с минусом). В результате такого соединения одна обкладка каждого конденсатора имеет одинаковый потенциал, например,, а другая. Разности потенциалов на обкладках всех конденсаторов при их параллельном соединении равны.

При параллельном соединении конденсаторов суммарная емкость соединения рассчитывается как сумма емкостей отдельных конденсаторов:

При параллельном соединении конденсаторов напряжение равно самой наименьшей величине рабочего напряжения конденсатора из состава рассматриваемого соединения.

Примеры решения задач

Понравился сайт? Расскажи друзьям!

Источник: http://ru.solverbook.com/spravochnik/fizika/posledovatelnoe-i-parallelnoe-soedinenie-kondensatorov/

Маркировка конденсаторов. Параллельное и последовательное соединение

Продолжаем обсуждение и изучение электронных компонентов под названием конденсаторы (ссылка). Основные аспекты устройства и принципа работы конденсаторов мы обсудили в предыдущей статье, а сегодня мы обсудим цифровую маркировку, а также разные варианты соединения конденсаторов. Сначала разберем теорию, а затем рассмотрим несколько практических примеров. Собственно, приступим к делу

Источник: https://microtechnics.ru/markirovka-kondensatorov-parallelnoe-i-posledovatelnoe-soedinenie/

Соединение конденсаторов Как правильно соединять конденсаторы?

 У многих начинающих любителей электроники в процессе сборки самодельного устройства возникает вопрос: “Как правильно соединять конденсаторы?”

 Казалось бы, зачем это надо, ведь если на принципиальной схеме указано, что в данном месте схемы должен быть установлен конденсатор на 47 микрофарад, значит, берём и ставим необходимый конденсатор. Но, согласитесь, что в мастерской даже заядлого электронщика может не оказаться конденсатора с необходимым номиналом!

Похожая ситуация может возникнуть и при ремонте какого-либо прибора. Например, необходим электролитический конденсатор ёмкостью 1000 микрофарад, а под рукой лишь 2 – 3 конденсатора на 470 микрофарад. Ставить конденсатор на 470 микрофарад, вместо положенных 1000? Нет, это допустимо не всегда. Так как же быть? Ехать на радиорынок за несколько десятков километров за одним конденсатором?

Важно

Как выйти из сложившейся ситуации? Можно соединить несколько конденсаторов и в результате получить необходимую нам ёмкость. В электронике существует два способа соединения конденсаторов: параллельное и последовательное.

В реальности это выглядит так:

Параллельное соединение

Принципиальная схема параллельного соединения

Последовательное соединение

Принципиальная схема последовательного соединения

Также можно комбинировать параллельное и последовательное соединение конденсаторов. На практике вам вряд ли это пригодиться.

Как рассчитать общую ёмкость соединённых конденсаторов?

Помогут нам в этом несколько простых формул. Не сомневайтесь, если вы будете заниматься электроникой, то эти простые формулы рано или поздно вас выручат.

Общая ёмкость параллельно соединённых конденсаторов:

С1 – ёмкость первого конденсатора;

С2 – ёмкость второго конденсатора;

С3 – ёмкость третьего конденсатора;

СN – ёмкость N-ого конденсатора;

Cобщ – суммарная ёмкость составного конденсатора.

Как видим, при параллельном соединении ёмкости конденсаторов нужно всего-навсего сложить!

Внимание! Все расчёты необходимо производить в одних единицах. Если рассчитываем ёмкости в микрофарадах, то нужно указывать ёмкость C1C2 в микрофарадах. Результат также получим в микрофарадах. Это правило стоит соблюдать, иначе ошибки не избежать!

Чтобы не допустить ошибку при переводе микрофарад в пикофарады или нанофарады можно воспользоваться специальной таблицей. В ней указаны приставки, используемые для краткой записи и множители, с помощью которых можно пересчитать значения величин. 

Ёмкость двух последовательно соединённых конденсаторов можно рассчитать по другой формуле. Она будет чуть сложнее:

Внимание! Данная формула справедлива только для двух конденсаторов! Для большего количества последовательно включенных конденсаторов потребуется другая формула. Она более запутанная, да и не всегда пригождается .

Или то же самое, но более понятно:

Если вы проведёте несколько расчётов, то увидите, что при последовательном соединении конденсаторов их результирующая ёмкость будет всегда меньше наименьшей ёмкости, включённой в данную цепочку. Что это значить? А это значит, что если соединить последовательно конденсатор ёмкостью 5, 100 и 35 пикофарад, то общая ёмкость составного конденсатора будет меньше 5.

Совет

В том случае, если для последовательного соединения применены конденсаторы одинаковой ёмкости, эта громоздкая формула упрощается и принимает вид:

Здесь, вместо буквы M ставиться количество конденсаторов, а C1 – ёмкость конденсатора.

Стоит также запомнить простое правило:

При последовательном соединении двух конденсаторов с одинаковой ёмкостью результирующая ёмкость будет в два раза меньше ёмкости каждого из конденсаторов.

Таким образом, если вы последовательно соедините два конденсатора, ёмкость каждого из которых 10 нанофарад, то в результате ёмкость составного конденсатора составит 5 нанофарад.

Проверим конденсатор, замерив ёмкость, и на практике подтвердим правильность показанных здесь формул для расчёта

Возьмём два плёночных конденсатора. Один на 15 нанофарад (0,015 мкф.), другой на 10 нанофарад (0,01 мкф.) Соединим их последовательно. Теперь возьмём мультиметр Victor VC9805+ с функцией измерения ёмкости конденсаторов и замерим суммарную ёмкость двух конденсаторов. Вот что мы получим (см. фото).

Замер ёмкости последовательно соединённых конденсаторов

Ёмкость составного конденсатора составила 6 нанофарад (0,006 мкф.)

А теперь проделаем то же самое, но для параллельного соединения конденсаторов. Проверим результат с помощью тестера (см. фото).

Измерение ёмкости параллельно соединённых конденсаторов

Как видим, при параллельном соединении ёмкость двух конденсаторов сложилась и составляет 25 нанофарад (0,025 мкф.).

Во-первых, не стоит забывать, что кроме ёмкости у конденсаторов есть ещё один немаловажный параметр, как номинальное напряжение.

При последовательном соединении конденсаторов напряжение между ними распределяется обратно пропорционально ёмкостям этих конденсаторов. Поэтому, есть смысл при последовательном соединении применять конденсаторы с номинальным напряжением равным тому, которое должно быть у конденсатора взамен которого мы ставим составной конденсатор.

Если же используются конденсаторы одинаковой ёмкости, то напряжение между ними разделится поровну.

Для электролитических конденсаторов.

При соединении электролитических конденсаторов строго соблюдайте полярность! При параллельном соединении электролитических конденсаторов всегда соединяйте минусовой вывод одного конденсатора с минусовым выводом другого. Плюсовой вывод с плюсовым.

Параллельное соединение электролитических конденсаторов

Схема параллельного соединения

Обратите внимание

В последовательном соединении электролитических конденсаторов ситуация обратная. Необходимо соединять плюсовой вывод с минусовым. Получается что-то вроде последовательного соединения батареек.

Последовательное соединение электролитических конденсаторов

Схема последовательного соединения конденсаторов

Также не забывайте про номинальное напряжение. При параллельном соединении каждый из задействованных конденсаторов должен иметь то номинальное напряжение, как если бы мы ставили в схему один конденсатор.

То есть если в схему нужно установить конденсатор с номинальным напряжением на 35 вольт и ёмкостью, например, 200 микрофарад, то взамен его можно параллельно соединить два конденсатора на 100 микрофарад и 35 вольт.

Если хоть один из этих конденсаторов будет иметь меньшее номинальное напряжение (например, 25 вольт), то он вскоре выйдет из строя.

Желательно, чтобы для составного конденсатора подбирались конденсаторы одного типа (плёночные, керамические, слюдяные, металлобумажные). Лучше будет, если они взяты из одной партии.

Конечно, возможно и смешанное (комбинированное) соединение конденсаторов, но в практике оно не применяется (я не видел ). Расчёт ёмкости смешанного соединения конденсаторов обычно достаётся тем, кто решает задачи по физике и сдаёт экзамены 🙂

Источник: http://radiodvor.com/news/fashion/soedinenie-kondensatorov-kak-pravilno-so.html

Последовательное соединение конденсаторов: формула :

Под последовательным соединением подразумевают случаи, когда два или больше элемента имеют вид цепи, при этом каждый из них соединяется с другим только в одной точке. Зачем конденсаторы так размещаются? Как это правильно сделать? Что необходимо знать? Какие особенности последовательное соединение конденсаторов имеет на практике? Какая формула результата?

Что необходимо знать для правильного соединения?

Увы, но здесь не всё так легко сделать, как может показаться. Многие новички думают, что если на схематическом рисунке написано, что необходим элемент на 49 микрофарад, то достаточно его просто взять и установить (или заменить равнозначным).

Но необходимые параметры подобрать сложно даже в профессиональной мастерской. И что делать, если нет нужных элементов? Допустим, есть такая ситуация: необходим конденсатор на 100 микрофарад, а есть несколько штук на 47. Поставить его не всегда можно.

Важно

Ехать на радиорынок за одним конденсатором? Не обязательно. Достаточно будет соединить пару элементов. Существует два основных способа: последовательное и параллельное соединение конденсаторов. Вот о первом мы и поговорим.

Но если говорить про последовательное соединение катушки и конденсатора, то тут особых проблем нет.

Зачем так делают?

Когда с ними проводятся такие манипуляции, то электрические заряды на обкладках отдельных элементов будут равны: КЕ=К1=К2=К3. КЕ – конечная емкость, К – пропускаемое значение конденсатора.

Почему так? Когда заряды поступают от источника питания на внешние обкладки, то на внутренних может быть осуществлен перенос величины, которая является значением элемента с наименьшими параметрами.

То есть если взять конденсатор на 3 мкФ, а после него подсоединить на 1 мкФ – то конечный результат будет 1 мкФ. Конечно, на первом можно будет наблюдать значение в 3 мкФ.

Но второй элемент не сможет столько пропустить, и он будет срезать всё, что больше необходимого значения, оставляя большую емкость на первоначальном конденсаторе. Давайте рассмотрим, что нужно рассчитать, когда делается последовательное соединение конденсаторов. Формула:

Н=КЕ/ОЕК

  • ОЕ – общая емкость;
  • Н – напряжение;
  • КЕ – конечная емкость.

Что ещё необходимо знать, чтобы правильно соединить конденсаторы?

Для начала не забывайте, что кроме ёмкости они ещё обладают номинальным напряжением. Почему? Когда осуществляется последовательное соединение, то напряжение распределяется обратно пропорционально их ёмкостям между ними самими.

Поэтому использовать такой подход имеет смысл только в тех случаях, когда любой конденсатор сможет предоставить минимально необходимые параметры работы. Если используются элементы, у которых одинаковая емкость, то напряжение между ними будет разделяться поровну.

Также небольшое предостережение относительно электролитических конденсаторов: при работе с ними всегда внимательно контролируйте их полярность. Ибо при игнорировании этого фактора последовательное соединение конденсаторов может дать ряд нежелательных эффектов. И хорошо, если всё ограничится только пробоем данных элементов.

Помните, что конденсаторы копят ток, и если что-то пойдёт не так, в зависимости от схемы может случиться прецедент, в результате которого из строя выйдут другие составляющие схемы.

Ток при последовательном соединении

Из-за того, что у него существует только один возможный путь протекания, он будет иметь одно значение для всех конденсаторов. При этом количество накопленного заряда везде обладает одинаковым значением. От емкости это не зависит.

Посмотрите на любую схему последовательного соединения конденсаторов. Правая обкладка первого соединена с левой второго и так далее. Если используется больше 1 элемента, то часть из них будет изолированной от общей цепи.

Таким образом, эффективная площадь обкладок становится меньшей и равняется параметрам самого маленького конденсатора. Какое физическое явление лежит в основе этого процесса? Дело в том, что как только конденсатор наполняется электрическим зарядом, то он перестаёт пропускать ток.

И он тогда не может протекать по всей цепи. Остальные конденсаторы в таком случае тоже не смогут заряжаться.

Падение напряженности и общая емкость

Каждый элемент понемногу рассеивает напряжение. Учитывая, что емкость ему обратно пропорциональна, то чем она меньше, тем большим будет падение.

Как уже упоминалось ранее, последовательно соединённые конденсаторы обладают одинаковым электрическим зарядом. Поэтому при делении всех выражений на общее значение можно получить уравнение, которое покажет всю емкость.

В этом последовательное и параллельное соединение конденсаторов сильно разнятся.

Пример № 1

Давайте воспользуемся представленными в статье формулами и рассчитаем несколько практических задач. Итак, у нас есть три конденсатора. Их емкость составляет: С1 = 25 мкФ, С2 = 30 мкФ и С3 = 20 мкФ. Они соединены последовательно.

Необходимо найти их общую емкость. Используем соответствующее уравнение 1/С: 1/С1 + 1/С2 + 1/С3 = 1/25 + 1/30 + 1/20 = 37/300.

Переводим в микрофарады, и общая емкость конденсатора при последовательном соединении (а группа в данном случае считается как один элемент) составляет примерно 8,11 мкФ.

Пример № 2

Давайте, чтобы закрепить наработки, решим ещё одну задачу. Имеется 100 конденсаторов. Емкость каждого элемента составляет 2 мкФ. Необходимо определить их общую емкость. Нужно их количество умножить на характеристику: 100*2=200 мкФ. Итак, общая емкость конденсатора при последовательном соединении составляет 200 микрофарад. Как видите, ничего сложного.

Заключение

Итак, мы проработали теоретические аспекты, разобрали формулы и особенности правильного соединения конденсаторов (последовательно) и даже решили несколько задачек. Хочется напомнить, чтобы читатели не упускали из внимания влияние номинального напряжения. Также желательно, чтобы подбирались элементы одного типа (слюдяные, керамические, металлобумажные, плёночные). Тогда последовательное соединение конденсаторов сможет дать нам наибольший полезный эффект.

Источник: https://www.syl.ru/article/237511/new_posledovatelnoe-soedinenie-kondensatorov-formula

Параллельное и последовательное соединение конденсаторов: способы, правила, формулы

Не всегда удаётся подобрать конденсатор нужного номинала

Очень часто начинающие домашние мастера, обнаружив поломку прибора, стараются самостоятельно обнаружить причину. Увидев сгоревшую деталь, они стараются найти подобную, а если это не удаётся, несут прибор в ремонт.

На самом деле, не обязательно, чтобы показатели совпадали. Можно использовать конденсаторы меньшего номинала, соединив их в цепь. Главное – сделать это правильно. При этом достигается сразу 3 цели – поломка устранена, приобретён опыт, сэкономлены средства семейного бюджета.

Попробуем разобраться, какие способы соединения существуют и на какие задачи рассчитаны последовательное и параллельное соединение конденсаторов.

Часто без соединения конденсаторов в батарею не обойтись. Главное – сделать это правильно

Соединение конденсаторов в батарею: способы выполнения

Существует 3 способа соединения, каждый из которых преследует свою определённую цель:

  1. Параллельное – выполняется в случае необходимости увеличить ёмкость, оставив напряжение на прежнем уровне.
  2. Последовательное – обратный эффект. Напряжение увеличивается, ёмкость уменьшается.
  3. Смешанное – увеличивается как ёмкость, так и напряжение.

Теперь рассмотрим каждый из способов более подробно.

Параллельное соединение: схемы, правила

На самом деле всё довольно просто. При параллельном соединении расчёт общей ёмкости можно вычислить путём простейшего сложения всех конденсаторов. Итоговая формула будет выглядеть следующим образом: Собщ= С₁ + С₂ + С₃ + … + Сn. При этом напряжение на каждом их элементов будет оставаться неизменным: Vобщ= V₁ = V₂ = V₃ = … = Vn.

Соединение при таком подключении будет иметь следующий вид:

Получается, что подобный монтаж подразумевает подключение всех пластин конденсаторов к точкам питания. Такой способ встречается наиболее часто. Но может произойти ситуация, когда важно увеличить напряжение. Разберёмся, каким образом это сделать.

Последовательное соединение: способ, используемый реже

При использовании способа последовательного подключения конденсаторов напряжение в цепи возрастает.

Оно складывается из напряжения всех элементов и выглядит так: Vобщ= V₁ + V₂ + V₃ +…+ Vn.

При этом ёмкость изменяется в обратной пропорции: 1/Собщ= 1/С₁ + 1/С₂ + 1/С₃ + … + 1/Сn. Рассмотрим изменения ёмкости и напряжения при последовательном включении на примере.

Дано: 3 конденсатора с напряжением 150 В и ёмкостью 300 мкф. Подключив их последовательно, получим:

  • напряжение: 150 + 150 + 150 = 450 В;
  • ёмкость: 1/300 + 1/300 + 1/300 = 1/С = 299 мкф.

Внешне подобное подключение обкладок (пластин) будет выглядеть так:

Выполняют такое соединение в том случае, если есть опасность пробоя диэлектрика конденсатора при подаче напряжения в цепь. Но ведь существует и ещё один способ монтажа.

Полезно знать! Применяют также последовательное и параллельное соединение резисторов и конденсаторов. Это делается с целью снижения подаваемого на конденсатор напряжения и исключения его пробоя. Однако следует учитывать, что напряжения должно быть достаточно для работы самого прибора.

Смешанное соединение конденсаторов: схема, причины необходимости применения

Такое подключение (его ещё называют последовательно-параллельным) применяют в случае необходимости увеличения, как ёмкости, так и напряжения. Здесь вычисление общих параметров немного сложнее, но не настолько, чтобы нельзя было разобраться начинающему радиолюбителю. Для начала посмотрим, как выглядит такая схема.

Составим алгоритм вычислений.

  • всю схему нужно разбить на отдельные части, высчитать параметры которых просто;
  • высчитываем номиналы;
  • вычисляем общие показатели, как при последовательном включении.

Выглядит подобный алгоритм следующим образом:

Преимущество смешанного включения конденсаторов в цепь по сравнению с последовательным или параллельным

Смешанное соединение конденсаторов решает задачи, которые не под силу параллельным и последовательным схемам. Его можно использовать при подключении электродвигателей либо иного оборудования, его монтаж возможен отдельными участками. Монтаж его намного проще за счёт возможности выполнения отдельными частями.

Интересно знать! Многие радиолюбители считают этот способ более простым и приемлемым, чем два предыдущих. На самом деле, так и есть, если полностью понять алгоритм действий и научиться пользоваться им правильно.

Смешанное, параллельное и последовательное соединение конденсаторов: на что обратить внимание при его выполнении

Соединяя конденсаторы, в особенности электролитические, обратите внимание на строгое соблюдение полярности. Параллельное присоединение подразумевает подключение «минус/минус», а последовательное – «плюс/минус». Все элементы должны быть однотипны –плёночные, керамические, слюдяные либо металлобумажные.

А вот что умеют делать всем известные китайские «изобретатели» – такой конденсатор явно долго не протянетПолезно знать! Выход из строя конденсаторов часто происходит по вине производителя, экономящего на деталях (чаще это приборы китайского производства). Поэтому правильно рассчитанные и собранные в схему элементы будут работать намного дольше. Конечно, при условии отсутствия замыкания в цепи, при котором работа конденсаторов невозможна в принципе.

Калькулятор расчёта ёмкости при последовательном соединении конденсаторов

А что делать, если необходимая ёмкость неизвестна? Не каждому хочется самостоятельно рассчитывать необходимую ёмкость конденсаторов вручную, а у кого-то на это просто нет времени. Для удобства производства подобных действий редакция Seti.

guru предлагает нашему уважаемому читателю воспользоваться онлайн-калькулятором расчёта конденсаторов при последовательном соединении или вычисления ёмкости. В работе он необычайно прост. Пользователю необходимо лишь ввести в поля необходимые данные, после чего нажать кнопку «Рассчитать».

Программы, в которые заложены все алгоритмы и формулы последовательного соединения конденсаторов, а также вычислений необходимой ёмкости, моментально выдаст необходимый результат.

Как рассчитать энергию заряженного конденсатора: выводим окончательную формулу

Первое, что для этого необходимо сделать – рассчитать, с какой силой притягиваются обкладки друг к другу. Это можно сделать по формуле F = q₀ × E, где q₀ является показателем величины заряда, а E – напряжённостью обкладок.

Далее нам необходим показатель напряжённости обкладок, который можно вычислить по формуле E = q / (2ε₀S), где q – заряд, ε₀ – постоянная величина, S – площадь обкладок.

В этом случае получим общую формулу для расчёта силы притяжения двух обкладок: F = q₂ / (2ε₀S).

Совет

Итогом наших умозаключений станет вывод выражения энергии заряженного конденсатора, как W = A = Fd. Однако это не окончательная формула, которая нам необходима.

Следуем далее: учитывая предыдущую информацию, мы имеем: W = dq₂ / (2ε₀S). При ёмкости конденсатора, выражаемой как C = d / (ε₀S) получаем результат W = q₂ / (2С).

Применив формулу q = СU, получим итог: W = CU² /2.

Редакция Seti.guru советует сохранить эту памятку

Конечно, для начинающего радиолюбителя все эти расчёты могут показаться сложными и непонятными, но при желании и некоторой усидчивости с ними можно разобраться. Вникнув в смысл, он поразится, насколько просто производятся все эти расчёты.

Для чего нужно знать показатель энергии конденсатора

По сути, расчёт энергии применяется редко, однако есть области, в которых это знать необходимо. К примеру, фотовспышка камеры – здесь вычисление показателя энергии очень важно. Она накапливается за определённое время (несколько секунд), а вот выдаётся мгновенно. Получается, что конденсатор сравним с аккумулятором – разница лишь в ёмкости.

Ни одна фотовспышка не сможет работать без накопителя энергии, такого, как конденсатор

Подводя итог

Порой без соединения конденсаторов не обойтись, ведь не всегда можно подобрать подходящие по номиналам.

Поэтому знание того как это сделать может выручить при поломке бытовой техники или электроники, что позволит значительно сэкономить на оплате труда специалиста по ремонту.

Как наверняка уже понял Уважаемый читатель, сделать это несложно и под силу даже начинающим домашним мастерам. А значит стоит потратить немного своего драгоценного времени и разобраться в алгоритме действий и правилах их выполнения.

Правильность соединения конденсаторов гарантирует их долгую бесперебойную работу

Надеемся, что информация, изложенная в сегодняшней статье, была полезна нашим читателям. Возможно, у Вас остались какие-либо вопросы? В этом случае их можно изложить в обсуждении ниже. Редакция Seti.guru с удовольствием на них ответит в максимально короткие сроки.

Если же Вы имеете опыт самостоятельного соединения конденсаторов (неважно, положительный он или отрицательный), убедительная просьба поделиться им с другими читателями. Это поможет начинающим мастерам более полно понять алгоритм действий и избежать ошибок. Пишите, делитесь, спрашивайте.

А напоследок мы предлагаем посмотреть короткий, но довольно информативный видеоролик по сегодняшней теме.

Источник: https://seti.guru/parallelnoe-i-posledovatelnoe-soedinenie-kondensatorov

Вывод формулы последовательного соединения конденсаторов

Как правильно соединять конденсаторы?

У многих начинающих любителей электроники в процессе сборки самодельного устройства возникает вопрос: “Как правильно соединять конденсаторы?”

Казалось бы, зачем это надо, ведь если на принципиальной схеме указано, что в данном месте схемы должен быть установлен конденсатор на 47 микрофарад, значит, берём и ставим. Но, согласитесь, что в мастерской даже заядлого электронщика может не оказаться конденсатора с необходимым номиналом!

Похожая ситуация может возникнуть и при ремонте какого-либо прибора. Например, необходим электролитический конденсатор ёмкостью 1000 микрофарад, а под рукой лишь два-три на 470 микрофарад. Ставить 470 микрофарад, вместо положенных 1000? Нет, это допустимо не всегда. Так как же быть? Ехать на радиорынок за несколько десятков километров и покупать недостающую деталь?

Как выйти из сложившейся ситуации? Можно соединить несколько конденсаторов и в результате получить необходимую нам ёмкость. В электронике существует два способа соединения конденсаторов: параллельное и последовательное.

В реальности это выглядит так:


Параллельное соединение


Принципиальная схема параллельного соединения


Последовательное соединение


Принципиальная схема последовательного соединения

Также можно комбинировать параллельное и последовательное соединение. Но на практике вам вряд ли это пригодиться.

Как рассчитать общую ёмкость соединённых конденсаторов?

Помогут нам в этом несколько простых формул. Не сомневайтесь, если вы будете заниматься электроникой, то эти простые формулы рано или поздно вас выручат.

Общая ёмкость параллельно соединённых конденсаторов:

С1 – ёмкость первого;

С2 – ёмкость второго;

С3 – ёмкость третьего;

СN – ёмкость N-ого конденсатора;

Cобщ – суммарная ёмкость составного конденсатора.

Как видим, при параллельном соединении ёмкости нужно всего-навсего сложить!

Внимание! Все расчёты необходимо производить в одних единицах. Если выполняем расчёты в микрофарадах, то нужно указывать ёмкость C1, C2 в микрофарадах. Результат также получим в микрофарадах. Это правило стоит соблюдать, иначе ошибки не избежать!

Чтобы не допустить ошибку при переводе микрофарад в пикофарады, а нанофарад в микрофарады, необходимо знать сокращённую запись численных величин. Также в этом вам поможет таблица. В ней указаны приставки, используемые для краткой записи и множители, с помощью которых можно производить пересчёт. Подробнее об этом читайте здесь.

Ёмкость двух последовательно соединённых конденсаторов можно рассчитать по другой формуле. Она будет чуть сложнее:

Внимание! Данная формула справедлива только для двух конденсаторов! Если их больше, то потребуется другая формула. Она более запутанная, да и на деле не всегда пригождается .

Или то же самое, но более понятно:

Если вы проведёте несколько расчётов, то увидите, что при последовательном соединении результирующая ёмкость будет всегда меньше наименьшей, включённой в данную цепочку. Что это значить? А это значит, что если соединить последовательно конденсаторы ёмкостью 5, 100 и 35 пикофарад, то общая ёмкость будет меньше 5.

В том случае, если для последовательного соединения применены конденсаторы одинаковой ёмкости, эта громоздкая формула волшебным образом упрощается и принимает вид:

Здесь, вместо буквы M ставиться количество конденсаторов, а C1 – его ёмкость.

Стоит также запомнить простое правило:

При последовательном соединении двух конденсаторов с одинаковой ёмкостью результирующая ёмкость будет в два раза меньше ёмкости каждого из них.

Таким образом, если вы последовательно соедините два конденсатора, ёмкость каждого из которых 10 нанофарад, то в результате она составит 5 нанофарад.

Не будем пускать слов по ветру, а проверим конденсатор, замерив ёмкость, и на практике подтвердим правильность показанных здесь формул.

Возьмём два плёночных конденсатора. Один на 15 нанофарад (0,015 мкф.),а другой на 10 нанофарад (0,01 мкф.) Соединим их последовательно. Теперь возьмём мультиметр Victor VC9805+ и замерим суммарную ёмкость двух конденсаторов. Вот что мы получим (см. фото).


Замер ёмкости при последовательном соединении

Ёмкость составного конденсатора составила 6 нанофарад (0,006 мкф. )

А теперь проделаем то же самое, но для параллельного соединения. Проверим результат с помощью того же тестера (см. фото).


Измерение ёмкости при параллельном соединении

Как видим, при параллельном соединении ёмкость двух конденсаторов сложилась и составляет 25 нанофарад (0,025 мкф.).

Что ещё необходимо знать, чтобы правильно соединять конденсаторы?

Во-первых, не стоит забывать, что есть ещё один немаловажный параметр, как номинальное напряжение.

При последовательном соединении конденсаторов напряжение между ними распределяется обратно пропорционально их ёмкостям. Поэтому, есть смысл при последовательном соединении применять конденсаторы с номинальным напряжением равным тому, которое имеет конденсатор, взамен которого мы ставим составной.

Если же используются конденсаторы с одинаковой ёмкостью, то напряжение между ними разделится поровну.

Для электролитических конденсаторов.

При соединении электролитических конденсаторов (электролитов) строго соблюдайте полярность! При параллельном соединении всегда подключайте минусовой вывод одного конденсатора к минусовому выводу другого,а плюсовой вывод с плюсовым.


Параллельное соединение электролитов


Схема параллельного соединения

В последовательном соединении электролитов ситуация обратная. Необходимо подключать плюсовой вывод к минусовому. Получается что-то вроде последовательного соединения батареек.


Последовательное соединение электролитов


Схема последовательного соединения

Также не забывайте про номинальное напряжение. При параллельном соединении каждый из задействованных конденсаторов должен иметь то номинальное напряжение, как если бы мы ставили в схему один конденсатор. То есть если в схему нужно установить конденсатор с номинальным напряжением на 35 вольт и ёмкостью, например, 200 микрофарад, то взамен его можно параллельно соединить два конденсатора на 100 микрофарад и 35 вольт. Если хоть один из них будет иметь меньшее номинальное напряжение (например, 25 вольт), то он вскоре выйдет из строя.

Желательно, чтобы для составного конденсатора подбирались конденсаторы одного типа (плёночные, керамические, слюдяные, металлобумажные). Лучше всего будет, если они взяты из одной партии, так как в таком случае разброс параметров у них будет небольшой.

Конечно, возможно и смешанное (комбинированное) соединение, но в практике оно не применяется (я не видел ). Расчёт ёмкости при смешанном соединении обычно достаётся тем, кто решает задачи по физике или сдаёт экзамены 🙂

Тем же, кто не на шутку увлёкся электроникой непременно надо знать, как правильно соединять резисторы и рассчитывать их общее сопротивление!

Схемы в электротехнике состоят из электрических элементов, в которых способы соединения конденсаторов могут быть разными. Надо понимать, как правильно подключить конденсатор. Отдельные участки цепи с подключенными конденсаторами можно заменить одним эквивалентным элементом. Он заменит ряд конденсаторов, но должно выполняться обязательное условие: когда напряжение, подводимое к обкладкам эквивалентного конденсатора, равняется напряжению на входе и выходе группы заменяющихся конденсаторов, тогда заряд емкости будет такой же, как и на группе емкостей. Для понимания вопроса, как подключить конденсатор в любой схеме, рассмотрим виды его включения.

Параллельное включение конденсаторов в цепь

Параллельное соединение конденсаторов — это когда все пластины подключаются к точкам включения цепи, образовывая батарею емкостей.

Параллельное соединение конденсаторов:

Разность потенциалов на пластинах накопителей емкости будет одинаковая, так как они все заряжаются от одного источника тока. В этом случае каждый заряжающийся конденсатор имеет собственный заряд при одинаковой величине, подводимой к ним энергии.

Параллельные конденсаторы, общий параметр количества заряда полученной батареи накопителей, рассчитывается, как сумма всех зарядов, помещающихся на каждой емкости, потому что каждый заряд емкости не зависит от заряда другой емкости, входящей в группу конденсаторов, параллельно включенных в схему.

При параллельном соединении конденсаторов емкость равняется:

Из представленной формулы можно сделать вывод, что всю группу накопителей можно рассматривать как один равноценный им конденсатор.

Конденсаторы, соединенные параллельно, имеют напряжение:

Последовательное включение конденсаторов в цепь

Когда в схеме выполнено последовательное соединение конденсаторов, оно выглядит как цепочка емкостных накопителей, где пластина первого и последнего накопителя емкости (конденсатора) подключены к источнику тока.

Последовательное соединение конденсатора:

При последовательном соединении конденсаторов все устройства этого участка берут одинаковое количество электроэнергии, потому что в процессе участвует первая и последняя пластинка накопителей, а пластины 2, 3 и другие до N проходят зарядку посредством влияния. По этой причине заряд пластины 2 накопителя емкости равняется по значению заряду 1 пластины, но имеет обратный знак. Заряд пластины накопителя 3 равняется значению заряда пластины 2, но так же с обратным знаком, все последующие накопители имеет аналогичную систему заряда.

Формула нахождения заряда на конденсаторе, схема подключения конденсатора:

Когда выполняется последовательное соединение конденсаторов, напряжение на каждом накопители емкости будет различное, так как в зарядке одинаковым количеством электрической энергии участвуют разные емкости. Зависимость емкости от напряжения такова: чем она меньше, тем большее напряжение необходимо подать на пластины накопителя для его зарядки. И обратная величина: чем выше емкость накопителя, тем меньше требуется напряжения для его зарядки. Можно сделать вывод, что емкость последовательно соединенных накопителей имеет значение для величины напряжения на пластинах — чем она меньше, тем больше напряжения требуется, а также накопители большой емкости требуют меньшего напряжения.

Основное отличие схемы последовательного соединения накопителей емкости в том, что электроэнергия протекает только в одном направлении, а это означает, что в каждом накопителе емкости составленной батареи ток будет одинаковым. В этом виде соединений конденсаторов обеспечивается равномерное накопление энергии независимо от емкости накопителей.

Группу накопителей емкости можно также на схеме рассматривать как эквивалентный накопитель, на пластины которого подается напряжение, определяемое формулой:

Заряд общего (эквивалентного) накопителя группы емкостных накопителей последовательного соединения равен:

Общему значению емкости последовательно соединенных конденсаторов соответствует выражение:

Смешанное включение емкостных накопителей в схему

Параллельное и последовательное соединение конденсаторов на одном из участков цепи схемы называется специалистами смешанным соединением.

Участок цепи подсоединенных смешанным включением накопителей емкости:

Смешанное соединение конденсаторов в схеме рассчитывается в определенном порядке, который можно представить следующим образом:

  • разбивается схема на простые для вычисления участки, это последовательное и параллельное соединение конденсаторов;
  • вычисляем эквивалентную емкость для группы конденсаторов, последовательно включенных на участке параллельного соединения;
  • проводим нахождение эквивалентной емкости на параллельном участке;
  • когда эквивалентные емкости накопителей определены, схему рекомендуется перерисовать;
  • рассчитывается емкость получившейся после последовательного включения эквивалентных накопителей электрической энергии.

Накопители емкостей (двухполюсники) включены разными способами в цепь, это дает несколько преимуществ в решении электротехнических задач по сравнению с традиционными способами включения конденсаторов:

  1. Использование для подключения электрических двигателей и другого оборудования в цехах, в радиотехнических устройствах.
  2. Упрощение вычисления величин электросхемы. Монтаж выполняется отдельными участками.
  3. Технические свойства всех элементов не меняются, когда изменяется сила тока и магнитное поле, это применяется для включения разных накопителей. Характеризуется постоянной величиной емкости и напряжения, а заряд пропорционален потенциалу.

Вывод

Разного вида включения конденсаторов в цепь применяются для решения электротехнических задач, в частности, для получения полярных накопителей из нескольких неполярных двухполюсников. В этом случае решением будет соединение группы однополюсных накопителей емкости по встречно-параллельному способу (треугольником). В этой схеме минус соединяется с минусом, а плюс — с плюсом. Происходит увеличение емкости накопителя, и меняется работа двухполюсника.

Не отображаются имеющиеся вхождения: последовательное параллельное и смешанное соединение конденсаторов, последовательное и параллельное соединение конденсаторов, при параллельном соединении конденсаторов емкость.

При параллельном соединении конденсаторов к каждому кон­денсатору приложено одинаковое напряжениеU, а величина за­ряда на обкладках каждого конденсатора Q пропорциональна его емкости (рис. 2).

Общий заряд Q всех конденсаторов

Общая емкость С, или емкость батареи, параллельно включенных конденсаторов равна сумме емкостей этих конденсаторов.

Параллельное подключение конденсатора к группе других включенных конденсаторов увеличивает общую емкость батареи этих конденсаторов. Следовательно, параллельное соединение конденсаторов при­меняется для увеличения емкости.

4)Если параллельно включены т одинаковых конденсаторов ем­костью С´ каждый, то общая (эквивалентная) емкость батареи этих конденсаторов может быть определена выражением

Последовательное соединение конденсаторов

На обкладках последовательно соединенных конденсаторов, подключенных к источнику постоянного тока с напряжением U, появятся заряды одинаковые по величине с противоположными знаками.

Напряжение на конденсаторах распределяется обратно пропорционально емкостям конденса­торов:

Обратная величина общей емкости последовательно соединенных конденсаторов равна сумме обратных величин емкостей этих кон­денсаторов.

При последовательном включении двух конденсаторов их об­щая емкость определяется следующим выражением:

Если в цепь включены последовательно п одинаковых конден­саторов емкостью С каждый, то общая емкость этих конденса­торов:

Из (14) видно, что, чем больше конденсаторов п соединено последовательно, тем меньше будет их общая емкость С, т. е. по­следовательное включение конденсаторов приводит к уменьше­нию общей емкости батареи конденсаторов.

На практике может оказаться , что допустимое ра­бочее напряжение Up конденсатора меньше напряжения, на кото­рое необходимо подключить конденсатор. Если этот конденсатор подключить на такое напряжение, то он выйдет из строя, так как будет пробит диэлектрик. Если же последовательно включить не­сколько конденсаторов, то напряжение распределится между ними и на каждом конденсаторе напряжение окажется мень­ше его допустимого рабочего Up. Следовательно, последовательное соединение конденсаторов применяют для того, чтобы напряжение на каждом конденсаторе не превышало его рабочего напряжения Up.

Смешанное соединение конденсаторов

Смешанное соединение (последовательно-параллельное) кон­денсаторов применяют тогда, когда необходимо увеличить ем­кость и рабочее напряжение батареи конденсаторов.

Рассмотрим смешанное соединение конденсаторов на ниже­приведенных примерах.

где Q — заряд конденсатора или конденсаторов, к которым при­ложено напряжение U; С — электрическая емкость конденсатора или батареи соединенных конденсаторов, к которой приложено напряжение U.

Таким образом, конденсаторы служат для накопления и сохра­нения электрического поля и его энергии.

15.Дайте определение понятиям трех лучевая звезда и треугольник сопротивлений. Запишите формулы для преобразования трех лучевой звезды сопротивлений в треугольник сопротивлений и наоборот. Преобразуйте схему к двум узлам (Рисунок 5)

Рисунок 5- Схема электрическая

Для облегчения расчета составляется схема замещения электрической цепи, т. е. схема, отображающая свойства цепи при определенных условиях.

На схеме замещения изображают все элементы, влиянием которых на результат расчета нельзя пренебречь, и указывают также электрические соединения между ними, которые имеются в цепи.

1.Схемы замещения элементов электрических цепей

На расчетных схемах источник энергии можно представить ЭДС без внутреннего сопротивления, если это сопротивление мало по сравнению с сопротивлением приемника (рис. 3.13,6).

Приr= 0 внутреннее падение напряженияUо = 0, поэтому

напряжение на зажимах источника при любом токе равно

В некоторых случаях источник электрической энергии на расчетной схеме заменяют другой (эквивалентной) схемой (рис. 3.14, а), где вместо ЭДСЕ источник характеризуется его током короткого замыканияIK, а вместо внутреннего со­противления в расчет вводится внутренняя проводимостьg=1/r.

Возможность такой замены можно доказать, разделив равенство (3.1) на r:

где U/r = Io—некоторый ток, равный отношению напряжения на зажимах источника к внутреннему сопротивлению;E/r = IK — ток короткого замыкания источника;

Вводя новые обозначения, получим равенство IK= Io + I, которому удовлетворяет эквивалентная схема рис. 3.14,а.

В этом случае при любой величине напряжения на зажимах; источника его ток остается равным току короткого замыкания (рис. 3.14,6):

Источник с неизменным током, не зависящим от внешнего сопротивления, называют источником тока.

Один и тот же источник электрической энергии может быть заменен в расчетной схеме источником ЭДС или источником тока.

Как соединить конденсаторы параллельным или последовательным соединением

Способы подключения конденсаторов в электрическую цепь

Схемы в электротехнике состоят из электрических элементов, в которых способы соединения конденсаторов могут быть разными. Надо понимать, как правильно подключить конденсатор. Отдельные участки цепи с подключенными конденсаторами можно заменить одним эквивалентным элементом.

Он заменит ряд конденсаторов, но должно выполняться обязательное условие: когда напряжение, подводимое к обкладкам эквивалентного конденсатора, равняется напряжению на входе и выходе группы заменяющихся конденсаторов, тогда заряд емкости будет такой же, как и на группе емкостей.

Для понимания вопроса, как подключить конденсатор в любой схеме, рассмотрим виды его включения.

Параллельное включение конденсаторов в цепь

Параллельное соединение конденсаторов — это когда все пластины подключаются к точкам включения цепи, образовывая батарею емкостей.

Параллельное соединение конденсаторов:

Параллельное соединение конденсаторов

Разность потенциалов на пластинах накопителей емкости будет одинаковая, так как они все заряжаются от одного источника тока. В этом случае каждый заряжающийся конденсатор имеет собственный заряд при одинаковой величине, подводимой к ним энергии.

Параллельные конденсаторы, общий параметр количества заряда полученной батареи накопителей, рассчитывается, как сумма всех зарядов, помещающихся на каждой емкости, потому что каждый заряд емкости не зависит от заряда другой емкости, входящей в группу конденсаторов, параллельно включенных в схему.

При параллельном соединении конденсаторов емкость равняется:

Формула и расшифровка

Из представленной формулы можно сделать вывод, что всю группу накопителей можно рассматривать как один равноценный им конденсатор.

Конденсаторы, соединенные параллельно, имеют напряжение:

Формула

Последовательное включение конденсаторов в цепь

Когда в схеме выполнено последовательное соединение конденсаторов, оно выглядит как цепочка емкостных накопителей, где пластина первого и последнего накопителя емкости (конденсатора) подключены к источнику тока.

Последовательное соединение конденсатора:

Формула

При последовательном соединении конденсаторов все устройства этого участка берут одинаковое количество электроэнергии, потому что в процессе участвует первая и последняя пластинка накопителей, а пластины 2, 3 и другие до N проходят зарядку посредством влияния.

По этой причине заряд пластины 2 накопителя емкости равняется по значению заряду 1 пластины, но имеет обратный знак.

Заряд пластины накопителя 3 равняется значению заряда пластины 2, но так же с обратным знаком, все последующие накопители имеет аналогичную систему заряда.

Формула нахождения заряда на конденсаторе, схема подключения конденсатора:

Последовательное соединение конденсаторов

Обратите внимание

Когда выполняется последовательное соединение конденсаторов, напряжение на каждом накопители емкости будет различное, так как в зарядке одинаковым количеством электрической энергии участвуют разные емкости.

Зависимость емкости от напряжения такова: чем она меньше, тем большее напряжение необходимо подать на пластины накопителя для его зарядки. И обратная величина: чем выше емкость накопителя, тем меньше требуется напряжения для его зарядки.

Можно сделать вывод, что емкость последовательно соединенных накопителей имеет значение для величины напряжения на пластинах — чем она меньше, тем больше напряжения требуется, а также накопители большой емкости требуют меньшего напряжения.

Основное отличие схемы последовательного соединения накопителей емкости в том, что электроэнергия протекает только в одном направлении, а это означает, что в каждом накопителе емкости составленной батареи ток будет одинаковым. В этом виде соединений конденсаторов обеспечивается равномерное накопление энергии независимо от емкости накопителей.

Группу накопителей емкости можно также на схеме рассматривать как эквивалентный накопитель, на пластины которого подается напряжение, определяемое формулой:

Основные моменты

Заряд общего (эквивалентного) накопителя группы емкостных накопителей последовательного соединения равен:

Формула

Общему значению емкости последовательно соединенных конденсаторов соответствует выражение:

Формула

Смешанное включение емкостных накопителей в схему

Параллельное и последовательное соединение конденсаторов на одном из участков цепи схемы называется специалистами смешанным соединением.

Участок цепи подсоединенных смешанным включением накопителей емкости:

Схема подключения конденсаторов

Смешанное соединение конденсаторов в схеме рассчитывается в определенном порядке, который можно представить следующим образом:

  • разбивается схема на простые для вычисления участки, это последовательное и параллельное соединение конденсаторов;
  • вычисляем эквивалентную емкость для группы конденсаторов, последовательно включенных на участке параллельного соединения;
  • проводим нахождение эквивалентной емкости на параллельном участке;
  • когда эквивалентные емкости накопителей определены, схему рекомендуется перерисовать;
  • рассчитывается емкость получившейся после последовательного включения эквивалентных накопителей электрической энергии.

Последовательное, параллельное и смешанное соединение конденсаторов

Накопители емкостей (двухполюсники) включены разными способами в цепь, это дает несколько преимуществ в решении электротехнических задач по сравнению с традиционными способами включения конденсаторов:

  1. Использование для подключения электрических двигателей и другого оборудования в цехах, в радиотехнических устройствах.
  2. Упрощение вычисления величин электросхемы. Монтаж выполняется отдельными участками.
  3. Технические свойства всех элементов не меняются, когда изменяется сила тока и магнитное поле, это применяется для включения разных накопителей. Характеризуется постоянной величиной емкости и напряжения, а заряд пропорционален потенциалу.

Вывод

Разного вида включения конденсаторов в цепь применяются для решения электротехнических задач, в частности, для получения полярных накопителей из нескольких неполярных двухполюсников.

В этом случае решением будет соединение группы однополюсных накопителей емкости по встречно-параллельному способу (треугольником). В этой схеме минус соединяется с минусом, а плюс — с плюсом.

Происходит увеличение емкости накопителя, и меняется работа двухполюсника.

Не отображаются имеющиеся вхождения: последовательное параллельное и смешанное соединение конденсаторов, последовательное и параллельное соединение конденсаторов, при параллельном соединении конденсаторов емкость.

Источник: https://domelectrik.ru/baza/komponenty/soedinenie-kondensatorov

Последовательное и параллельное соединение конденсаторов

Для достижения нужной емкости или при напряжении, превышающем номинальное напряжение, конденсаторы, могут соединяться последовательно или параллельно. Любое же сложное соединение состоит из нескольких комбинаций последовательного и параллельного соединений.

Последовательное соединение конденсаторов

При последовательном соединении, конденсаторы подключены таким образом, что только первый и последний конденсатор подключены к источнику ЭДС/тока одной из своих пластин.

Заряд одинаков на всех пластинах, но внешние заряжаются от источника, а внутренние образуются только за счет разделения зарядов ранее нейтрализовавших друг друга.

При этом заряд конденсаторов в батарее меньше, чем, если бы каждый конденсатор подключался бы отдельно. Следовательно, и общая емкость батареи конденсаторов меньше.

Напряжение на данном участке цепи соотносятся следующим образом:

Зная, что напряжение конденсатора можно представить через заряд и емкость, запишем:

Сократив выражение на Q, получим знакомую формулу:

Откуда эквивалентная емкость батареи конденсаторов соединенных последовательно:

Параллельное соединение конденсаторов

При параллельном соединении конденсаторов напряжение на обкладках одинаковое, а заряды разные.

Величина общего заряда полученного конденсаторами, равна сумме зарядов всех параллельно подключенных конденсаторов. В случае батареи из двух конденсаторов:

Так как заряд конденсатора

А напряжения на каждом из конденсаторов равны, получаем следующее выражение для эквивалентной емкости двух параллельно соединенных конденсаторов

Пример 1

Важно

Какова результирующая емкость 4 конденсаторов включенных последовательно и параллельно, если известно что С1 = 10 мкФ, C2 = 2 мкФ, C3 = 5 мкФ, а C4 = 1 мкФ?

При последовательном соединении общая емкость равна:

При параллельном соединении общая емкость равна:

Пример 2

Определить результирующую емкость группы конденсаторов подключенных последовательно-параллельно, если известно, что С1 = 7 мкФ, С2 = 2 мкФ, С3 = 1 мкФ.

Сначала найдем общую емкость параллельного участка цепи:

Затем найдем общую емкость для всей цепи:

По сути, расчет общей емкости конденсаторов схож с расчетом общего сопротивления цепи в случае с последовательным или параллельным соединением, но при этом, зеркально противоположен.

Советуем прочесть — Заряд и разряд конденсатора

1 1 1 1 1 1 1 1 1 1 4.50 (1 Голос)

Источник: https://electroandi.ru/elektronika/posledovatelnoe-i-parallelnoe-soedinenie-kondensatorov.html

Параллельное и последовательное соединение конденсаторов: способы, правила, формулы

Любая электроника в доме может выйти из строя. Однако сразу бежать в сервис не стоит – простейшие приборы может продиагностировать и починить даже начинающий радиолюбитель. К примеру, сгоревший конденсатор виден невооружённым глазом.

Но как быть, если под рукой нет детали подходящего номинала? Конечно, соединить 2 и более в цепь.

Сегодня поговорим о таких понятиях, как параллельное и последовательное соединение конденсаторов, разберемся, как его выполнить, узнаем о способах соединения, правилах его выполнения.

Не всегда удаётся подобрать конденсатор нужного номинала

Очень часто начинающие домашние мастера, обнаружив поломку прибора, стараются самостоятельно обнаружить причину. Увидев сгоревшую деталь, они стараются найти подобную, а если это не удаётся, несут прибор в ремонт.

На самом деле, не обязательно, чтобы показатели совпадали. Можно использовать конденсаторы меньшего номинала, соединив их в цепь. Главное – сделать это правильно. При этом достигается сразу 3 цели – поломка устранена, приобретён опыт, сэкономлены средства семейного бюджета.

Попробуем разобраться, какие способы соединения существуют и на какие задачи рассчитаны последовательное и параллельное соединение конденсаторов.

Часто без соединения конденсаторов в батарею не обойтись. Главное – сделать это правильно

Соединение конденсаторов в батарею: способы выполнения

Существует 3 способа соединения, каждый из которых преследует свою определённую цель:

  1. Параллельное – выполняется в случае необходимости увеличить ёмкость, оставив напряжение на прежнем уровне.
  2. Последовательное – обратный эффект. Напряжение увеличивается, ёмкость уменьшается.
  3. Смешанное – увеличивается как ёмкость, так и напряжение.

Теперь рассмотрим каждый из способов более подробно.

Параллельное соединение: схемы, правила

На самом деле всё довольно просто. При параллельном соединении расчёт общей ёмкости можно вычислить путём простейшего сложения всех конденсаторов. Итоговая формула будет выглядеть следующим образом: Собщ= С₁ + С₂ + С₃ + … + Сn. При этом напряжение на каждом их элементов будет оставаться неизменным: Vобщ= V₁ = V₂ = V₃ = … = Vn.

Соединение при таком подключении будет иметь следующий вид:

Получается, что подобный монтаж подразумевает подключение всех пластин конденсаторов к точкам питания. Такой способ встречается наиболее часто. Но может произойти ситуация, когда важно увеличить напряжение. Разберёмся, каким образом это сделать.

Последовательное соединение: способ, используемый реже

При использовании способа последовательного подключения конденсаторов напряжение в цепи возрастает.

Оно складывается из напряжения всех элементов и выглядит так: Vобщ= V₁ + V₂ + V₃ +…+ Vn.

При этом ёмкость изменяется в обратной пропорции: 1/Собщ= 1/С₁ + 1/С₂ + 1/С₃ + … + 1/Сn. Рассмотрим изменения ёмкости и напряжения при последовательном включении на примере.

Дано: 3 конденсатора с напряжением 150 В и ёмкостью 300 мкф. Подключив их последовательно, получим:

  • напряжение: 150 + 150 + 150 = 450 В;
  • ёмкость: 1/300 + 1/300 + 1/300 = 1/С = 299 мкф.

Внешне подобное подключение обкладок (пластин) будет выглядеть так:

Выполняют такое соединение в том случае, если есть опасность пробоя диэлектрика конденсатора при подаче напряжения в цепь. Но ведь существует и ещё один способ монтажа.

Полезно знать! Применяют также последовательное и параллельное соединение резисторов и конденсаторов. Это делается с целью снижения подаваемого на конденсатор напряжения и исключения его пробоя. Однако следует учитывать, что напряжения должно быть достаточно для работы самого прибора.

Смешанное соединение конденсаторов: схема, причины необходимости применения

Такое подключение (его ещё называют последовательно-параллельным) применяют в случае необходимости увеличения, как ёмкости, так и напряжения. Здесь вычисление общих параметров немного сложнее, но не настолько, чтобы нельзя было разобраться начинающему радиолюбителю. Для начала посмотрим, как выглядит такая схема.

Составим алгоритм вычислений.

  • всю схему нужно разбить на отдельные части, высчитать параметры которых просто;
  • высчитываем номиналы;
  • вычисляем общие показатели, как при последовательном включении.

Выглядит подобный алгоритм следующим образом:

Преимущество смешанного включения конденсаторов в цепь по сравнению с последовательным или параллельным

Смешанное соединение конденсаторов решает задачи, которые не под силу параллельным и последовательным схемам. Его можно использовать при подключении электродвигателей либо иного оборудования, его монтаж возможен отдельными участками. Монтаж его намного проще за счёт возможности выполнения отдельными частями.

Интересно знать! Многие радиолюбители считают этот способ более простым и приемлемым, чем два предыдущих. На самом деле, так и есть, если полностью понять алгоритм действий и научиться пользоваться им правильно.

Смешанное, параллельное и последовательное соединение конденсаторов: на что обратить внимание при его выполнении

Соединяя конденсаторы, в особенности электролитические, обратите внимание на строгое соблюдение полярности. Параллельное присоединение подразумевает подключение «минус/минус», а последовательное – «плюс/минус». Все элементы должны быть однотипны –плёночные, керамические, слюдяные либо металлобумажные.

А вот что умеют делать всем известные китайские «изобретатели» – такой конденсатор явно долго не протянетПолезно знать! Выход из строя конденсаторов часто происходит по вине производителя, экономящего на деталях (чаще это приборы китайского производства). Поэтому правильно рассчитанные и собранные в схему элементы будут работать намного дольше. Конечно, при условии отсутствия замыкания в цепи, при котором работа конденсаторов невозможна в принципе.

Калькулятор расчёта ёмкости при последовательном соединении конденсаторов

А что делать, если необходимая ёмкость неизвестна? Не каждому хочется самостоятельно рассчитывать необходимую ёмкость конденсаторов вручную, а у кого-то на это просто нет времени. Для удобства производства подобных действий редакция Seti.

guru предлагает нашему уважаемому читателю воспользоваться онлайн-калькулятором расчёта конденсаторов при последовательном соединении или вычисления ёмкости. В работе он необычайно прост. Пользователю необходимо лишь ввести в поля необходимые данные, после чего нажать кнопку «Рассчитать».

Программы, в которые заложены все алгоритмы и формулы последовательного соединения конденсаторов, а также вычислений необходимой ёмкости, моментально выдаст необходимый результат.

Как рассчитать энергию заряженного конденсатора: выводим окончательную формулу

Первое, что для этого необходимо сделать – рассчитать, с какой силой притягиваются обкладки друг к другу. Это можно сделать по формуле F = q₀ × E, где q₀ является показателем величины заряда, а E – напряжённостью обкладок.

Далее нам необходим показатель напряжённости обкладок, который можно вычислить по формуле E = q / (2ε₀S), где q – заряд, ε₀ – постоянная величина, S – площадь обкладок.

В этом случае получим общую формулу для расчёта силы притяжения двух обкладок: F = q₂ / (2ε₀S).

Совет

Итогом наших умозаключений станет вывод выражения энергии заряженного конденсатора, как W = A = Fd. Однако это не окончательная формула, которая нам необходима.

Следуем далее: учитывая предыдущую информацию, мы имеем: W = dq₂ / (2ε₀S). При ёмкости конденсатора, выражаемой как C = d / (ε₀S) получаем результат W = q₂ / (2С).

Применив формулу q = СU, получим итог: W = CU² /2.

Редакция Seti.guru советует сохранить эту памятку

Конечно, для начинающего радиолюбителя все эти расчёты могут показаться сложными и непонятными, но при желании и некоторой усидчивости с ними можно разобраться. Вникнув в смысл, он поразится, насколько просто производятся все эти расчёты.

Для чего нужно знать показатель энергии конденсатора

По сути, расчёт энергии применяется редко, однако есть области, в которых это знать необходимо. К примеру, фотовспышка камеры – здесь вычисление показателя энергии очень важно. Она накапливается за определённое время (несколько секунд), а вот выдаётся мгновенно. Получается, что конденсатор сравним с аккумулятором – разница лишь в ёмкости.

Ни одна фотовспышка не сможет работать без накопителя энергии, такого, как конденсатор

Подводя итог

Порой без соединения конденсаторов не обойтись, ведь не всегда можно подобрать подходящие по номиналам.

Поэтому знание того как это сделать может выручить при поломке бытовой техники или электроники, что позволит значительно сэкономить на оплате труда специалиста по ремонту.

Как наверняка уже понял Уважаемый читатель, сделать это несложно и под силу даже начинающим домашним мастерам. А значит стоит потратить немного своего драгоценного времени и разобраться в алгоритме действий и правилах их выполнения.

Правильность соединения конденсаторов гарантирует их долгую бесперебойную работу

Надеемся, что информация, изложенная в сегодняшней статье, была полезна нашим читателям. Возможно, у Вас остались какие-либо вопросы? В этом случае их можно изложить в обсуждении ниже. Редакция Seti.guru с удовольствием на них ответит в максимально короткие сроки.

Если же Вы имеете опыт самостоятельного соединения конденсаторов (неважно, положительный он или отрицательный), убедительная просьба поделиться им с другими читателями. Это поможет начинающим мастерам более полно понять алгоритм действий и избежать ошибок. Пишите, делитесь, спрашивайте.

А напоследок мы предлагаем посмотреть короткий, но довольно информативный видеоролик по сегодняшней теме.

Источник: https://seti.guru/parallelnoe-i-posledovatelnoe-soedinenie-kondensatorov

Соединение конденсаторов: последовательное, параллельное и смешанное

В электротехнике существуют различные варианты подключения электрических элементов. В частности, существует последовательное, параллельное или смешанное соединение конденсаторов, в зависимости от потребностей схемы. Рассмотрим их.

Параллельное соединение

Параллельное соединение характеризуется тем, что все пластины электрических конденсаторов присоединяются к точкам включения и образовывают собой батареи. В таком случае, во время заряда конденсаторов каждый из них будет иметь различное число электрических зарядов при одинаковом количестве подводимой энергии

Схема параллельного крепления

Емкость при параллельной установке рассчитывается исходя из емкостей всех конденсаторов в схеме. При этом, количество электрической энергии, поступающей на все отдельные двухполюсные элементы цепи, можно будет рассчитать, суммировав сумму энергии, помещающейся в каждый конденсатор. Вся схема, подключенная таким образом, рассчитывается как один двухполюсник.

Cобщ = C1 + C2 + C3

Схема – напряжение на накопителях

В отличие от соединения звездой, на обкладки всех конденсаторов попадает одинаковое напряжение. Например, на схеме выше мы видим, что:

VAB = VC1 = VC2 = VC3 = 20 Вольт

Последовательное соединение

Здесь к точкам включения присоединяются контакты только первого и последнего конденсатора.

Схема – схема последовательного соединения

Главной особенностью работы схемы является то, что электрическая энергия будет проходить только по одному направлению, значит, что в каждом из конденсаторов ток будет одинаковым.

В такой цепи для каждого накопителя, независимо от его емкости, будет обеспечиваться равное накопление проходящей энергии.

Обратите внимание

Нужно понимать, что каждый из них последовательно соприкасается со следующим и предыдущим, а значит, емкость при последовательном типе может воспроизводиться энергией соседнего накопителя.

Формула, которая отражает зависимость тока от соединения конденсаторов, имеет такой вид:

i = ic1 = ic2 = ic3 = ic4, то есть токи проходящие через каждый конденсатор равны между собой.

Следовательно, одинаковой будет не только сила тока, но и электрический заряд. По формуле это определяется как:

Qобщ= Q1 = Q2 = Q3

А так определяется общая суммарная емкость конденсаторов при последовательном соединении:

1/Cобщ = 1/C1 + 1/C2 + 1/C3

Видео: как соединять конденсаторы параллельным и последовательным методом

Смешанное подключение

Но, стоит учитывать, что для соединения различных конденсаторов необходимо учитывать напряжение сети. Для каждого полупроводника этот показатель будет отличаться в зависимости от емкости элемента.

Отсюда следует, что отдельные группы полупроводниковых двухполюсников малой емкости будут при зарядке становиться больше, и наоборот, электроемкость большого размера будет нуждаться в меньшем заряде.

Схема: смешанное соединение конденсаторов

Существует также смешанное соединение двух и более конденсаторов. Здесь электрическая энергия распределяется одновременно при помощи параллельного и последовательного подключения электролитических элементов в цепь.

Эта схема имеет несколько участков с различным подключением конденсирующих двухполюсников. Иными словами, на одном цепь параллельно включена, на другом – последовательно.

Такая электрическая схема имеет ряд достоинств сравнительно с традиционными:

  1. Можно использовать для любых целей: подключения электродвигателя, станочного оборудования, радиотехнических приборов;
  2. Простой расчет. Для монтажа вся схема разбивается на отдельные участки цепи, которые рассчитываются по отдельности;
  3. Свойства компонентов не изменяются независимо от изменений электромагнитного поля, силы тока. Это очень важно при работе с разноименными двухполюсниками. Ёмкость постоянна при постоянном напряжении, но, при этом, потенциал пропорционален заряду;
  4. Если требуется собрать несколько неполярных полупроводниковых двухполюсников из полярных, то нужно взять несколько однополюсных двухполюсника и соединить их встречно-параллельным способом (в треугольник). Минус к минусу, а плюс к плюсу. Таким образом, за счет увеличения емкости изменяется принцип работы двухполюсного полупроводника.

Источник: https://www.asutpp.ru/soedinenie-kondensatorov.html

Соединение конденсаторов

Радиоэлектроника для начинающих

У многих начинающих любителей электроники в процессе сборки самодельного устройства возникает вопрос: “Как правильно соединять конденсаторы?”

Казалось бы, зачем это надо, ведь если на принципиальной схеме указано, что в данном месте схемы должен быть установлен конденсатор на 47 микрофарад, значит, берём и ставим. Но, согласитесь, что в мастерской даже заядлого электронщика может не оказаться конденсатора с необходимым номиналом!

Похожая ситуация может возникнуть и при ремонте какого-либо прибора. Например, необходим электролитический конденсатор ёмкостью 1000 микрофарад, а под рукой лишь два-три на 470 микрофарад. Ставить 470 микрофарад, вместо положенных 1000? Нет, это допустимо не всегда. Так как же быть? Ехать на радиорынок за несколько десятков километров и покупать недостающую деталь?

Важно

Как выйти из сложившейся ситуации? Можно соединить несколько конденсаторов и в результате получить необходимую нам ёмкость. В электронике существует два способа соединения конденсаторов: параллельное и последовательное.

В реальности это выглядит так:

Параллельное соединение

Принципиальная схема параллельного соединения

Последовательное соединение

Принципиальная схема последовательного соединения

Также можно комбинировать параллельное и последовательное соединение. Но на практике вам вряд ли это пригодиться.

Как рассчитать общую ёмкость соединённых конденсаторов?

Помогут нам в этом несколько простых формул. Не сомневайтесь, если вы будете заниматься электроникой, то эти простые формулы рано или поздно вас выручат.

Общая ёмкость параллельно соединённых конденсаторов:

С1 – ёмкость первого;

С2 – ёмкость второго;

С3 – ёмкость третьего;

СN – ёмкость N-ого конденсатора;

Cобщ – суммарная ёмкость составного конденсатора.

Как видим, при параллельном соединении ёмкости нужно всего-навсего сложить!

Внимание! Все расчёты необходимо производить в одних единицах. Если выполняем расчёты в микрофарадах, то нужно указывать ёмкость C1, C2 в микрофарадах. Результат также получим в микрофарадах. Это правило стоит соблюдать, иначе ошибки не избежать!

Чтобы не допустить ошибку при переводе микрофарад в пикофарады, а нанофарад в микрофарады, необходимо знать сокращённую запись численных величин. Также в этом вам поможет таблица. В ней указаны приставки, используемые для краткой записи и множители, с помощью которых можно производить пересчёт. Подробнее об этом читайте здесь.

Ёмкость двух последовательно соединённых конденсаторов можно рассчитать по другой формуле. Она будет чуть сложнее:

Внимание! Данная формула справедлива только для двух конденсаторов! Если их больше, то потребуется другая формула. Она более запутанная, да и на деле не всегда пригождается .

Или то же самое, но более понятно:

Если вы проведёте несколько расчётов, то увидите, что при последовательном соединении результирующая ёмкость будет всегда меньше наименьшей, включённой в данную цепочку. Что это значить? А это значит, что если соединить последовательно конденсаторы ёмкостью 5, 100 и 35 пикофарад, то общая ёмкость будет меньше 5.

В том случае, если для последовательного соединения применены конденсаторы одинаковой ёмкости, эта громоздкая формула волшебным образом упрощается и принимает вид:

Здесь, вместо буквы M ставиться количество конденсаторов, а C1 – его ёмкость.

Стоит также запомнить простое правило:

При последовательном соединении двух конденсаторов с одинаковой ёмкостью результирующая ёмкость будет в два раза меньше ёмкости каждого из них.

Совет

Таким образом, если вы последовательно соедините два конденсатора, ёмкость каждого из которых 10 нанофарад, то в результате она составит 5 нанофарад.

Не будем пускать слов по ветру, а проверим конденсатор, замерив ёмкость, и на практике подтвердим правильность показанных здесь формул.

Возьмём два плёночных конденсатора. Один на 15 нанофарад (0,015 мкф.),а другой на 10 нанофарад (0,01 мкф.) Соединим их последовательно. Теперь возьмём мультиметр Victor VC9805+ и замерим суммарную ёмкость двух конденсаторов. Вот что мы получим (см. фото).

Замер ёмкости при последовательном соединении

Ёмкость составного конденсатора составила 6 нанофарад (0,006 мкф.)

А теперь проделаем то же самое, но для параллельного соединения. Проверим результат с помощью того же тестера (см. фото).

Измерение ёмкости при параллельном соединении

Как видим, при параллельном соединении ёмкость двух конденсаторов сложилась и составляет 25 нанофарад (0,025 мкф.).

Во-первых, не стоит забывать, что есть ещё один немаловажный параметр, как номинальное напряжение.

При последовательном соединении конденсаторов напряжение между ними распределяется обратно пропорционально их ёмкостям. Поэтому, есть смысл при последовательном соединении применять конденсаторы с номинальным напряжением равным тому, которое имеет конденсатор, взамен которого мы ставим составной.

Если же используются конденсаторы с одинаковой ёмкостью, то напряжение между ними разделится поровну.

Для электролитических конденсаторов

При соединении электролитических конденсаторов (электролитов) строго соблюдайте полярность! При параллельном соединении всегда подключайте минусовой вывод одного конденсатора к минусовому выводу другого,а плюсовой вывод с плюсовым.

Параллельное соединение электролитов

Схема параллельного соединения

В последовательном соединении электролитов ситуация обратная. Необходимо подключать плюсовой вывод к минусовому. Получается что-то вроде последовательного соединения батареек.

Последовательное соединение электролитов

Схема последовательного соединения

Также не забывайте про номинальное напряжение. При параллельном соединении каждый из задействованных конденсаторов должен иметь то номинальное напряжение, как если бы мы ставили в схему один конденсатор.

Обратите внимание

То есть если в схему нужно установить конденсатор с номинальным напряжением на 35 вольт и ёмкостью, например, 200 микрофарад, то взамен его можно параллельно соединить два конденсатора на 100 микрофарад и 35 вольт.

Если хоть один из них будет иметь меньшее номинальное напряжение (например, 25 вольт), то он вскоре выйдет из строя.

Желательно, чтобы для составного конденсатора подбирались конденсаторы одного типа (плёночные, керамические, слюдяные, металлобумажные). Лучше всего будет, если они взяты из одной партии, так как в таком случае разброс параметров у них будет небольшой.

Конечно, возможно и смешанное (комбинированное) соединение, но в практике оно не применяется (я не видел ). Расчёт ёмкости при смешанном соединении обычно достаётся тем, кто решает задачи по физике или сдаёт экзамены 🙂

Тем же, кто не на шутку увлёкся электроникой непременно надо знать, как правильно соединять резисторы и рассчитывать их общее сопротивление!

Главная » Радиоэлектроника для начинающих » Текущая страница

Также Вам будет интересно узнать:

Источник: http://go-radio.ru/connection-of-capacitors.html

Как соединить конденсаторы

Июль 23, 2014

12796 просмотров

В предыдущих статьях были рассмотрены вопросы работы и характеристики конденсаторов. Сейчас Я расскажу о всех методах соединения конденсаторов для подключения в схему. Сразу скажу, что в жизни практически везде, за исключением редких случаев используется только параллельная схема подключения.

Следует знать, что в цепи переменного тока конденсатор выступает еще как емкостное сопротивление. При чем с увеличением величины емкости конденсатора- уменьшается сопротивление в цепи переменного тока.

Параллельное соединение конденсаторов

При параллельной схеме подключения все обкладки конденсаторов соединяются в две группы, причем один вывод с каждого конденсатора соединяется в одну группу с другими, а второй — в другую.

Наглядный пример параллельного соединения и схема на картинке.
Все параллельно соединенные конденсаторы подключаются к одному источнику напряжения, поэтому существует на них две точки разности потенциалов или напряжения.

На всех выводах конденсаторов будет абсолютно одинаковое напряжение.

При  подключении параллельно все конденсаторы вместе, образуют принципиально одну емкость, величина которой будет равняться сумме всех емкостей подключенных в цепи конденсаторов.При параллельном подключении через каждый из конденсаторов потечет разный ток, который будет зависеть от величины емкости каждого из них. Чем выше емкость, тем больший ток потечет через неё.

Параллельное соединение очень часто встречается в жизни. С его помощью можно из группы конденсаторов собрать любую необходимую емкость.

Например, для запуска 3 фазного электродвигателя в однофазной сети 220 Вольт в результате расчетов Вы получили что необходима рабочая емкость 125 мкФ. Такой емкости конденсаторов Вы не найдете в продаже.

Для того, что бы получить необходимую емкость придется купить и соединить параллельно 3 конденсатора один на 100 мкФ, второй- на 20, и третий на 5 мкФ.

Соединение конденсаторов последовательно

При последовательном соединении конденсаторов каждая из обкладок соединяется только в одной точке с одной обкладкой другого кон­денсатора. Получается цепочка конденсаторов.

Крайние два вывода подключаются к источнику тока, в результате чего происходит перераспределение между ними электрических зарядов.

Заряды на всех промежуточных обкладках одинаковые величине с чередованием по знаку. 

Через все соединенные конденсаторы последовательно протекает одинаковой величины ток, потому что у него нет другого пути прохождения.

Общая же емкость будет ограничиваться площадью обкладок самого маленького по величине, потому что как только зарядится полностью конденсатор с самой маленькой емкостью- вся цепочка перестанет пропускать ток и заряд остальных прервется.

Высчитывается же емкость по этой формуле:Но при последовательном соединении увеличивается расстояние (или изоляция) между обкладками до величины равной сумме расстояний между обкладками всех последовательно подключенных конденсаторов.

Важно

Например, если взять два конденсатора с рабочим напряжением 200 Вольт и соединить последовательно, то изоляция между их обкладками сможет выдержать 1000 Вольт при подключении в схему.

Из выше сказанного можно сделать вывод, что последовательно соединять необходимо:

  1. Для получения эквивалентного меньшего по емкости конденсатора.
  2. Если необходима емкость, работающая на более высоких напряжениях.
  3. Для создания емкостного делителя напряжения, который позволяет получить меньшей величины напряжение из более высокого.

Практически, для получения первого и второго достаточно просто купить один конденсатор с необходимой величиной емкости или рабочим напряжением. Поэтому данный метод соединения в жизни не встречается.

Смешанное соединение конденсаторов

Встречается смешанное соединение только на различных платах. Для него характерно наличие в одной цепи параллельного и последовательного соединения конденсаторов. При чем смешанное соединение может быть как последовательного, так параллельного характера.

В жизни подробные знания о смешанном соединении могут только пригодится радиолюбителям, поэтому не буду на этом подробно останавливаться.

Из следующей статьи Вы узнаете как правильно проверить и определить емкость конденсатора.

Источник: http://jelektro.ru/elektricheskie-terminy/soedinenie-kondensatorov.html

Последовательное и параллельное соединение. Применение и схемы

В электрических цепях элементы могут соединяться по различным схемам, в том числе они имеют последовательное и параллельное соединение.

Последовательное соединение

При таком соединении проводники соединяются друг с другом последовательно, то есть, начало одного проводника будет соединяться с концом другого.

Основная особенность данного соединения заключается в том, что все проводники принадлежат одному проводу, нет никаких разветвлений. Через каждый из проводников будет протекать один и тот же электрический ток.

Но суммарное напряжение на проводниках будет равняться вместе взятым напряжениям на каждом из них.

Рассмотрим некоторое количество резисторов, соединенных последовательно. Так как нет разветвлений, то количество проходящего заряда через один проводник, будет равно количеству заряда, прошедшего через другой проводник. Силы тока на всех проводниках будут одинаковыми. Это основная особенность данного соединения.

Это соединение можно рассмотреть иначе. Все резисторы можно заменить одним эквивалентным резистором.

Ток на эквивалентном резисторе будет совпадать с общим током, протекающим через все резисторы. Эквивалентное общее напряжение будет складываться из напряжений на каждом резисторе. Это является разностью потенциалов на резисторе.

Если воспользоваться этими правилами и законом Ома, который подходит для каждого резистора, можно доказать, что сопротивление эквивалентного общего резистора будет равно сумме сопротивлений. Следствием первых двух правил будет являться третье правило.

Применение

Последовательное соединение используется, когда нужно целенаправленно включать или выключать какой-либо прибор, выключатель соединяют с ним по последовательной схеме. Например, электрический звонок будет звенеть только тогда, когда он будет последовательно соединен с источником и кнопкой.

Согласно первому правилу, если электрический ток отсутствует хотя бы на одном из проводников, то его не будет и на других проводниках. И наоборот, если ток имеется хотя бы на одном проводнике, то он будет и на всех других проводниках. Также работает карманный фонарик, в котором есть кнопка, батарейка и лампочка.

Все эти элементы необходимо соединить последовательно, так как нужно, чтобы фонарик светил, когда будет нажата кнопка.

Иногда последовательное соединение не приводит к нужным целям.

Совет

Например, в квартире, где много люстр, лампочек и других устройств, не следует все лампы и устройства соединять последовательно, так как никогда не требуется одновременно включать свет в каждой из комнат квартиры.

Для этого последовательное и параллельное соединение рассматривают отдельно, и для подключения осветительных приборов в квартире применяют параллельный вид схемы.

Параллельное соединение

В этом виде схемы все проводники соединяются параллельно друг с другом. Все начала проводников объединены в одну точку, и все концы также соединены вместе. Рассмотрим некоторое количество однородных проводников (резисторов), соединенных по параллельной схеме.

Этот вид соединения является разветвленным. В каждой ветви содержится по одному резистору. Электрический ток, дойдя до точки разветвления, разделяется на каждый резистор, и будет равняться сумме токов на всех сопротивлениях. Напряжение на всех элементах, соединенных параллельно, является одинаковым.

Все резисторы можно заменить одним эквивалентным резистором. Если воспользоваться законом Ома, можно получить выражение сопротивления. Если при последовательном соединении сопротивления складывались, то при параллельном будут складываться величины обратные им, как записано в формуле выше.

Применение

Если рассматривать соединения в бытовых условиях, то в квартире лампы освещения, люстры должны быть соединены параллельно.

Если их соединить последовательно, то при включении одной лампочки мы включим все остальные.

При параллельном же соединении мы можем, добавляя соответствующий выключатель в каждую из ветвей, включать соответствующую лампочку по мере желания. При этом такое включение одной лампы не влияет на остальные лампы.

Все электрические бытовые устройства в квартире соединены параллельно в сеть с напряжением 220 В, и подключены к распределительному щитку. Другими словами, параллельное соединение используется при необходимости подключения электрических устройств независимо друг от друга. Последовательное и параллельное соединение имеют свои особенности. Существуют также смешанные соединения.

Работа тока

Последовательное и параллельное соединение, рассмотренное ранее, было справедливо для величин напряжения, сопротивления и силы тока, являющихся основными. Работа тока определяется по формуле:

А = I х U х t, где А – работа тока, t – время течения по проводнику.

Для определения работы при последовательной схеме соединения, необходимо заменить в первоначальном выражении напряжение. Получаем:

А=I х (U1 + U2) х t

Раскрываем скобки и получаем, что на всей схеме работа определяется суммой на каждой нагрузке.

Обратите внимание

Точно также рассматриваем параллельную схему соединения. Только меняем уже не напряжение, а силу тока. Получается результат:

А = А1+А2

Мощность тока

При рассмотрении формулы мощности участка цепи снова необходимо пользоваться формулой:

Р=U х I

После аналогичных рассуждений выходит результат, что последовательное и параллельное соединение можно определить следующей формулой мощности:

Р=Р1 + Р2

Другими словами, при любых схемах общая мощность равна сумме всех мощностей в схеме. Этим можно объяснить, что не рекомендуется включать в квартире сразу несколько мощных электрических устройств, так как проводка может не выдержать такой мощности.

Влияние схемы соединения на новогоднюю гирлянду

После перегорания одной лампы в гирлянде можно определить вид схемы соединения. Если схема последовательная, то не будет гореть ни одной лампочки, так как сгоревшая лампочка разрывает общую цепь. Чтобы выяснить, какая именно лампочка сгорела, нужно проверять все подряд. Далее, заменить неисправную лампу, гирлянда будет функционировать.

При применении параллельной схемы соединения гирлянда будет продолжать работать, даже если одна или несколько ламп сгорели, так как цепь не разорвана полностью, а только один небольшой параллельный участок. Для восстановления такой гирлянды достаточно увидеть, какие лампы не горят, и заменить их.

Последовательное и параллельное соединение для конденсаторов

При последовательной схеме возникает такая картина: заряды от положительного полюса источника питания идут только на наружные пластины крайних конденсаторов.

Конденсаторы, находящиеся между ними, передают заряд по цепи. Этим объясняется появление на всех пластинах равных зарядов с разными знаками.

Исходя из этого, заряд любого конденсатора, соединенного по последовательной схеме, можно выразить такой формулой:

qобщ= q1 = q2 = q3

Для определения напряжения на любом конденсаторе, необходима формула:

U= q/С

Где С — емкость. Суммарное напряжение выражается таким же законом, который подходит для сопротивлений. Поэтому получаем формулу емкости:

С= q/(U1 + U2 + U3)

Чтобы сделать эту формулу проще, можно перевернуть дроби и заменить отношение разности потенциалов к заряду емкости. В результате получаем:

1/С= 1/С1 + 1/С2 + 1/C3

Немного иначе рассчитывается параллельное соединение конденсаторов.

Общий заряд вычисляется как сумма всех зарядов, накопившихся на пластинах всех конденсаторов. А величина напряжения также вычисляется по общим законам. В связи с этим формула суммарной емкости при параллельной схеме соединения выглядит так:

С= (q1 + q2 + q3)/U

Это значение рассчитывается как сумма каждого прибора в схеме:

С=С1 + С2 + С3

Смешанное соединение проводников

В электрической схеме участки цепи могут иметь и последовательное и параллельное соединение, переплетающихся между собой. Но все законы, рассмотренные выше для отдельных видов соединений, справедливы по-прежнему, и используются по этапам.

Сначала нужно мысленно разложить схему на отдельные части. Для лучшего представления ее рисуют на бумаге. Рассмотрим наш пример по изображенной выше схеме.

Удобнее всего ее изобразить, начиная с точек Б и В. Они расставляются на некотором расстоянии между собой и от края листа бумаги. С левой стороны к точке Б подключается один провод, а справа отходят два провода. Точка В наоборот, слева имеет две ветки, а после точки отходит один провод.

Важно

Далее нужно изобразить пространство между точками. По верхнему проводнику расположены 3 сопротивления с условными значениями 2, 3, 4. Снизу будет идти ток с индексом 5. Первые 3 сопротивления включены в схему последовательно, а пятый резистор подключен параллельно.

Остальные два сопротивления (первый и шестой) подключены последовательно с рассматриваемым нами участком Б-В. Поэтому схему дополняем 2-мя прямоугольниками по сторонам от выбранных точек.

Теперь используем формулу расчета сопротивления:

  • Первая формула для последовательного вида соединения.
  • Далее, для параллельной схемы.
  • И окончательно для последовательной схемы.

Аналогичным образом можно разложить на отдельные схемы любую сложную схему, включая соединения не только проводников в виде сопротивлений, но и конденсаторов.

Чтобы научиться владеть приемами расчета по разным видам схем, необходимо потренироваться на практике, выполнив несколько заданий.

Похожие темы:

Источник: https://electrosam.ru/glavnaja/jelektrotehnika/raschjoty/posledovatelnoe-i-parallelnoe-soedinenie/

Определение ёмкости последовательно или параллельно соединённых конденсаторов – формула

Практически все электрические цепи включают в себя емкостные элементы. Соединение конденсаторов между собой выполняют по схемам. Их необходимо знать как при расчетах, так и при выполнении монтажа.

Последовательное соединение

Конденсатор, а в просторечии – «ёмкость», та деталь, без которой не обходится ни одна электрическая или электронная плата. Даже в современных гаджетах он присутствует, правда, уже в измененном виде.

Вспомним, что представляет собой этот радиотехнический элемент. Это накопитель электрических зарядов и энергии, 2 проводящие пластины, между которыми расположен диэлектрик. При прикладывании к пластинам источника постоянного тока через устройство кратковременно потечет ток, и оно зарядится до напряжения источника. Его емкость используют для решения технических задач.

Само это слово произошло задолго до того, как придумали устройство. Термин появился ещё тогда, когда люди считали, что электричество – это что-то типа жидкости, и ею можно наполнить какой-нибудь сосуд. Применительно к конденсатору – он неудачен, т.к. подразумевает, что прибор может вместить только конечное количество электричества. Хотя это и не так, но термин остался неизменным.

Чем больше пластины, и меньше расстояние между ними, тем больше ёмкость конденсатора. Если его обкладки соединить с каким-либо проводником, то через этот проводник произойдет быстрый разряд.

В координатных телефонных станциях с помощью этой особенности происходит обмен сигналами между приборами. Длина импульсов, необходимых для команд, таких как: «соединение линии», «ответ абонента», «отбой», регулируется величиной ёмкости установленных в цепь конденсаторов.

Единица измерения ёмкости – 1 Фарад. Т.к. это большая величина, то пользуются микрофарадами, пикофарадами и нанофарадами, (мкФ, пФ, нФ).

На практике, выполнив последовательное соединение, можно добиться увеличения прикладываемого напряжения. В этом случае поданное напряжение получают 2 внешние обкладки собранной системы, а обкладки, находящиеся внутри, заряжаются с помощью распределения зарядов. К таким приемам прибегают, когда под рукой не оказывается нужных элементов, зато есть детали других номиналов по напряжению.

К участку, имеющему 2 последовательно соединенных конденсатора, рассчитанных на напряжение 125 В, можно подключить питание 250 В.

Совет

Если для постоянного тока, конденсатор является препятствием за счет своего диэлектрического промежутка, то с переменным – все иначе. Для токов разных частот, подобно катушкам и резисторам, сопротивление конденсатора будет меняться. Токи высокой частоты он пропускает хорошо, а для их собратьев низкой частоты создает барьер.

У радиолюбителей есть способ – через емкость 220-500 пФ к радиоприемнику подключают вместо антенны сеть освещения напряжением 220 В. Ток с частотой 50 Гц он отфильтрует, а токи высокой частоты пропустит. Это сопротивление конденсаторов легко рассчитать по формуле для емкостного сопротивления:RC =1/6*f*C.

Где:

  • Rc – емкостное сопротивление, Ом;
  • f – частота тока, Гц;
  • C – емкость данного конденсатора, Ф;
  • 6 – округленное до целой части число 2π.

Но не только прикладываемое напряжение к цепи можно изменить, пользуясь подобной схемой включения. Так добиваются изменений емкостей при последовательных соединениях. Для легкости запоминания придумали подсказку, что общее значение емкости, полученное при выборе подобной схемы, получается всегда меньше меньшей из двух, включенных в цепочку.

Если так соединить 2 детали одинаковой ёмкости, то их общее значение будет вдвое меньше каждой из них. Расчеты последовательных соединений конденсатора можно выполнить по приведенной ниже формуле:

Собщ = С1*С2/С1+С2,

Пусть С1=110 пФ, а С2=220 пФ, тогда Собщ = 110×220/110+220 = 73 пФ.

Не стоит забывать про простоту и удобство монтажа, а также обеспечение качественной работы собранного устройства или оборудования. В последовательных соединениях у емкостей должен быть 1 производитель. А если детали всей цепочки будут одной партии выпуска, то проблем с эксплуатацией созданной цепи не будет.

Параллельное соединение

Накопители электрического заряда постоянной емкости, различают:

  • керамические;
  • бумажные;
  • слюдяные;
  • металлобумажные;
  • электролитические конденсаторы.

Источник: https://odinelectric.ru/knowledgebase/opredelenie-emkosti-posledovatelno-parallelno-soedinennyh-kondensatorov

Параллельное и последовательное соединение конденсаторов: способы, правила, формулы

Любая электроника в доме может выйти из строя. Однако сразу бежать в сервис не стоит – простейшие приборы может продиагностировать и починить даже начинающий радиолюбитель. К примеру, сгоревший конденсатор виден невооружённым глазом. Но как быть, если под рукой нет детали подходящего номинала? Конечно, соединить 2 и более в цепь. Сегодня поговорим о таких понятиях, как параллельное и последовательное соединение конденсаторов, разберемся, как его выполнить, узнаем о способах соединения, правилах его выполнения.

Нет конденсатора нужного номинала: что делать

Очень часто начинающие домашние мастера, обнаружив поломку прибора, стараются самостоятельно обнаружить причину. Увидев сгоревшую деталь, они стараются найти подобную, а если это не удаётся, несут прибор в ремонт. На самом деле, не обязательно, чтобы показатели совпадали. Можно использовать конденсаторы меньшего номинала, соединив их в цепь. Главное – сделать это правильно. При этом достигается сразу 3 цели – поломка устранена, приобретён опыт, сэкономлены средства семейного бюджета.

Попробуем разобраться, какие способы соединения существуют и на какие задачи рассчитаны последовательное и параллельное соединение конденсаторов.

Соединение конденсаторов в батарею: способы выполнения

Существует 3 способа соединения, каждый из которых преследует свою определённую цель:

  1. Параллельное – выполняется в случае необходимости увеличить ёмкость, оставив напряжение на прежнем уровне.
  2. Последовательное – обратный эффект. Напряжение увеличивается, ёмкость уменьшается.
  3. Смешанное – увеличивается как ёмкость, так и напряжение.

Теперь рассмотрим каждый из способов более подробно.

Параллельное соединение: схемы, правила

На самом деле всё довольно просто. При параллельном соединении расчёт общей ёмкости можно вычислить путём простейшего сложения всех конденсаторов. Итоговая формула будет выглядеть следующим образом: Собщ= С₁ + С₂ + С₃ + … + Сn. При этом напряжение на каждом их элементов будет оставаться неизменным: Vобщ= V₁ = V₂ = V₃ = … = Vn.

Соединение при таком подключении будет иметь следующий вид:

Получается, что подобный монтаж подразумевает подключение всех пластин конденсаторов к точкам питания. Такой способ встречается наиболее часто. Но может произойти ситуация, когда важно увеличить напряжение. Разберёмся, каким образом это сделать.

Последовательное соединение: способ, используемый реже

При использовании способа последовательного подключения конденсаторов напряжение в цепи возрастает. Оно складывается из напряжения всех элементов и выглядит так: Vобщ= V₁ + V₂ + V₃ +…+ Vn. При этом ёмкость изменяется в обратной пропорции: 1/Собщ= 1/С₁ + 1/С₂ + 1/С₃ + … + 1/Сn. Рассмотрим изменения ёмкости и напряжения при последовательном включении на примере.

Дано: 3 конденсатора с напряжением 150 В и ёмкостью 300 мкф. Подключив их последовательно, получим:

  • напряжение: 150 + 150 + 150 = 450 В,
  • ёмкость: 1/300 + 1/300 + 1/300 = 1/С = 299 мкф.

Внешне подобное подключение обкладок (пластин) будет выглядеть так:

Выполняют такое соединение в том случае, если есть опасность пробоя диэлектрика конденсатора при подаче напряжения в цепь. Но ведь существует и ещё один способ монтажа.

Полезно знать! Применяют также последовательное и параллельное соединение резисторов и конденсаторов. Это делается с целью снижения подаваемого на конденсатор напряжения и исключения его пробоя. Однако следует учитывать, что напряжения должно быть достаточно для работы самого прибора.

Смешанное соединение конденсаторов: схема, причины необходимости применения

Такое подключение (его ещё называют последовательно-параллельным) применяют в случае необходимости увеличения, как ёмкости, так и напряжения. Здесь вычисление общих параметров немного сложнее, но не настолько, чтобы нельзя было разобраться начинающему радиолюбителю. Для начала посмотрим, как выглядит такая схема.

Составим алгоритм вычислений.

  • всю схему нужно разбить на отдельные части, высчитать параметры которых просто,
  • высчитываем номиналы,
  • вычисляем общие показатели, как при последовательном включении.

Выглядит подобный алгоритм следующим образом:

Преимущество смешанного включения конденсаторов в цепь по сравнению с последовательным или параллельным

Смешанное соединение конденсаторов решает задачи, которые не под силу параллельным и последовательным схемам. Его можно использовать при подключении электродвигателей либо иного оборудования, его монтаж возможен отдельными участками. Монтаж его намного проще за счёт возможности выполнения отдельными частями.

Интересно знать! Многие радиолюбители считают этот способ более простым и приемлемым, чем два предыдущих. На самом деле, так и есть, если полностью понять алгоритм действий и научиться пользоваться им правильно.

Смешанное, параллельное и последовательное соединение конденсаторов: на что обратить внимание при его выполнении

Соединяя конденсаторы, в особенности электролитические, обратите внимание на строгое соблюдение полярности. Параллельное присоединение подразумевает подключение «минус/минус», а последовательное – «плюс/минус». Все элементы должны быть однотипны –плёночные, керамические, слюдяные либо металлобумажные.

Полезно знать! Выход из строя конденсаторов часто происходит по вине производителя, экономящего на деталях (чаще это приборы китайского производства). Поэтому правильно рассчитанные и собранные в схему элементы будут работать намного дольше. Конечно, при условии отсутствия замыкания в цепи, при котором работа конденсаторов невозможна в принципе.

Калькулятор расчёта ёмкости при последовательном соединении конденсаторов

А что делать, если необходимая ёмкость неизвестна? Не каждому хочется самостоятельно рассчитывать необходимую ёмкость конденсаторов вручную, а у кого-то на это просто нет времени. Для удобства производства подобных действий редакция Seti.guru предлагает нашему уважаемому читателю воспользоваться онлайн-калькулятором расчёта конденсаторов при последовательном соединении или вычисления ёмкости. В работе он необычайно прост. Пользователю необходимо лишь ввести в поля необходимые данные, после чего нажать кнопку «Рассчитать». Программы, в которые заложены все алгоритмы и формулы последовательного соединения конденсаторов, а также вычислений необходимой ёмкости, моментально выдаст необходимый результат.

Как рассчитать энергию заряженного конденсатора: выводим окончательную формулу

Первое, что для этого необходимо сделать – рассчитать, с какой силой притягиваются обкладки друг к другу. Это можно сделать по формуле F = q₀ × E, где q₀ является показателем величины заряда, а E – напряжённостью обкладок. Далее нам необходим показатель напряжённости обкладок, который можно вычислить по формуле E = q / (2ε₀S), где q – заряд, ε₀ – постоянная величина, S – площадь обкладок. В этом случае получим общую формулу для расчёта силы притяжения двух обкладок: F = q₂ / (2ε₀S).

Итогом наших умозаключений станет вывод выражения энергии заряженного конденсатора, как W = A = Fd. Однако это не окончательная формула, которая нам необходима. Следуем далее: учитывая предыдущую информацию, мы имеем: W = dq₂ / (2ε₀S). При ёмкости конденсатора, выражаемой как C = d / (ε₀S) получаем результат W = q₂ / (2С). Применив формулу q = СU, получим итог: W = CU² /2.

Конечно, для начинающего радиолюбителя все эти расчёты могут показаться сложными и непонятными, но при желании и некоторой усидчивости с ними можно разобраться. Вникнув в смысл, он поразится, насколько просто производятся все эти расчёты.

Для чего нужно знать показатель энергии конденсатора

По сути, расчёт энергии применяется редко, однако есть области, в которых это знать необходимо. К примеру, фотовспышка камеры – здесь вычисление показателя энергии очень важно. Она накапливается за определённое время (несколько секунд), а вот выдаётся мгновенно. Получается, что конденсатор сравним с аккумулятором – разница лишь в ёмкости.

Подводя итог

Порой без соединения конденсаторов не обойтись, ведь не всегда можно подобрать подходящие по номиналам. Поэтому знание того как это сделать может выручить при поломке бытовой техники или электроники, что позволит значительно сэкономить на оплате труда специалиста по ремонту. Как наверняка уже понял Уважаемый читатель, сделать это несложно и под силу даже начинающим домашним мастерам. А значит стоит потратить немного своего драгоценного времени и разобраться в алгоритме действий и правилах их выполнения.

Надеемся, что информация, изложенная в сегодняшней статье, была полезна нашим читателям. Возможно, у Вас остались какие-либо вопросы? В этом случае их можно изложить в обсуждении ниже. Редакция Seti.guru с удовольствием на них ответит в максимально короткие сроки. Если же Вы имеете опыт самостоятельного соединения конденсаторов (неважно, положительный он или отрицательный), убедительная просьба поделиться им с другими читателями. Это поможет начинающим мастерам более полно понять алгоритм действий и избежать ошибок. Пишите, делитесь, спрашивайте. А напоследок мы предлагаем посмотреть короткий, но довольно информативный видеоролик по сегодняшней теме.

Загрузка…

Последовательное, параллельное и смешанное соединение конденсаторов

Лекция № 4

Тема 1. 2. Электрические цепи постоянного тока. Электрические измерения

План

  1. Последовательное, параллельное и смешанное соединение конденсаторов

1. В электрических цепях применяются различные способы соединения конденсаторов. Соединение конденсаторов может производиться: после -довательно, параллельно и последовательно-параллельно (последнее иногда называют смешанное соединение конденсаторов). Существующие виды соединения конденсаторов показаны на рисунке 1.

Если группа конденсаторов включена в цепь таким образом, что к точкам включения непосредственно присоединены пластины всех конденсаторов, то такое соединение называется параллельным соединением конденсаторов (рисунок 2.).

Рисунок 2. Параллельное соединение конденсаторов.

При заряде группы конденсаторов, соединенных параллельно, между пластинами всех конденсаторов будет одна и та же разность потенциалов, так как все они заряжаются от одного и того же источника тока. Общее же количество электричества на всех конденсаторах будет равно сумме количеств электричества, помещающихся на каждом из конденсаторов, так как заряд каждого их конденсаторов происходит независимо от заряда других конденсаторов данной группы. Исходя из этого, всю систему параллельно соединенных конденсаторов можно рассматривать как один эквивалентный (равноценный) конденсатор. Тогда общая емкость конденсаторов при параллельном соединении равна сумме емкостей всех соединенных конденсаторов.

Обозначим суммарную емкость соединенных в батарею конденсаторов буквой Собщ, емкость первого конденсатора С1 емкость второго С2 и емкость третьего С3. Тогда для параллельного соединения конденсаторов будет справедлива следующая формула:

Последний знак + и многоточие указывают на то, что этой формулой можно пользоваться при четырех, пяти и вообще при любом числе конденсаторов.

Если же соединение конденсаторов в батарею производится в виде цепочки и к точкам включения в цепь непосредственно присоединены пластины только первого и последнего конденсаторов, то такое соединение конденсаторов называется последовательным (рисунок 3).

Рисунок 2. Последовательное соединение конденсаторов.

При последовательном соединении все конденсаторы заряжаются одинаковым количеством электричества, так как непосредственно от источника тока заряжаются только крайние пластины (1 и 6), а остальные пластины (2, 3, 4 и 5) заряжаются через влияние. При этом заряд пластины 2 будет равен по величине и противоположен по знаку заряду пластины 1, заряд пластины 3 будет равен по величине и противоположен по знаку заряду пластины 2 и т. д.

Напряжения на различных конденсаторах будут, вообще говоря, различными, так как для заряда одним и тем же количеством электричества конденсаторов различной емкости всегда требуются различные напряжения. Чем меньше емкость конденсатора, тем большее напряжение необходимо для того, чтобы зарядить этот конденсатор требуемым количеством электричества, и наоборот.

Таким образом, при заряде группы конденсаторов, соединенных последовательно, на конденсаторах малой емкости напряжения будут больше, а на конденсаторах большой емкости — меньше.

Аналогично предыдущему случаю можно рассматривать всю группу конденсаторов, соединенных последовательно, как один эквивалентный конденсатор, между пластинами которого существует напряжение, равное сумме напряжений на всех конденсаторах группы, а заряд которого равен заряду любого из конденсаторов группы.

Возьмем самый маленький конденсатор в группе. На нем должно быть самое большое напряжение. Но напряжение на этом конденсаторе составляет только часть общего напряжения, существующего на всей группе конденсаторов. Напряжение на всей группе больше напряжения на конденсаторе, имеющем самую малую емкость. А отсюда непосредственно следует, что общая емкость группы конденсаторов, соединенных последовательно, меньше емкости самого малого конденсатора в группе.

Для вычисления общей емкости при последовательном соединении конденсаторов удобнее всего пользоваться следующей формулой:

Для частного случая двух последовательно соединенных конденсаторов формула для вычисления их общей емкости будет иметь вид:

Последовательно-параллельным соединением конденса-торов называется цепь имеющая в своем составе участки, как с параллельным, так и с последовательным соединением конденсаторов.

На рисунке 4 приведен пример участка цепи со смешанным соединением конденсаторов.

Рисунок 4. Последовательно-параллельное соединение конденсаторов.

При расчете общей емкости такого участка цепи с последовательно-параллельным соединением конденсаторов этот участок разбивают на простейшие участки, состоящие только из групп с последовательным или параллельным соединением конденсаторов. Дальше алгоритм расчета имеет вид:

1. Определяют эквивалентную емкость участков с последовательным соединением конденсаторов.

2. Если эти участки содержат последовательно соединенные конденсаторы, то сначала вычисляют их емкость.

3. После расчета эквивалентных емкостей конденсаторов перерисовывают схему. Обычно получается цепь из последовательно соединенных эквивалентных конденсаторов.

4. Рассчитывают емкость полученной схемы.

Один из примеров расчета емкости при смешанном соединении конденсаторов приведен на рисунке 5.

Вопросы для самопроверки:

  1. Перечислить классификацию конденсаторов.

  2. Объяснить, чему равна энергия заряженного конденсатора.

  3. Перечислить способы соединения конденсаторов применяются в электрических цепях.

  4. Обьяснить, какой способ соединения конденсаторов  наз. параллельным?

  5. Определить, чему равна суммарная емкость конденсаторов при параллельном соединении.

Последовательное и параллельное соединение конденсаторов

Конденсатор (от лат. — «уплотнение») — пассивный элемент, предназначенный для накопления заряда в электрической цепи, состоящий из двух обкладок, которые разделены слоем диэлектрика. Емкость измеряется в Фарадах — Ф.

Последовательное соединение конденсаторов

Рисунок — Последовательное соединение конденсаторов

При последовательном соединение конденсаторов, напряжение суммируется, заряды равны, а ёмкость определяется по формуле, то есть:

Uобщ=U1+U2+U3

qобщ=q1=q2=q3


Параллельное соединение конденсаторов

Рисунок — Параллельном соединение конденсаторов

При параллельном соединение конденсаторов ёмкость и заряд складываются, а напряжения равны между собой то есть:

Собщ123

qобщ=q1+q2+q3

Uобщ=U1=U2=U3

Электроёмкость конденсатора рассчитывается по формуле:
Формула ёмкости плоского конденсатора:

Формула энергии заряженного конденсатора:

C – электроёмкость, Ф;
q – заряд, Кл;
U – напряжение между обкладками, В;
φ12– разность потенциалов, В;
W — энергии заряженного конденсатора, Дж;
S – площадь пластин, м2;
d – расстояние между пластинами, м;
ε0 – электрическая постоянная, 8,85∙10-12 Ф/м;
ε – относительная диэлектрическая постоянная, Ф/м;
Формула для определения напряженности одной пластины конденсатора:

E/2


Обозначение емкости конденсаторов

μF, uF, mF — микрофарады 10-6 мкФ

nF — нанофарады 10-6 нФ

pF, mmF, uuF — пикофарады 10-6 пФ


Таблица маркировки конденсаторов

Заряд при последовательном соединении | Gadget-apple.ru

Отдельные конденсаторы могут быть соединены друг с другом различным образом. При этом во всех случаях можно найти емкость некоторого равнозначного конденсатора, который может заменить ряд соединенных между собой конденсаторов.

Для равнозначного конденсатора выполняется условие: если подводимое к обкладкам равнозначного конденсатора напряжение равно напряжению, подводимому к крайним зажимам группы конденсаторов, то равнозначный конденсатор накопит такой же заряд, как и группа конденсаторов.

Параллельное соединение конденсаторов

На рис. 1 изображено параллельное соединение нескольких конденсаторов. В этом случае напряжения, подводимые к отдельным конденсаторам, одинаковы: U1 = U2 = U3 = U. Заряды на обкладках отдельных конденсаторов: Q1 = C1U , Q 2 = C 2 U , Q 3 = C 3 U , а заряд, полученный от источника Q = Q1 + Q2 + Q3.

Рис. 1. Схема параллельного соединения конденсаторов

Общая емкость равнозначного (эквивалентного) конденсатора:

C = Q / U = (Q1 + Q2 + Q3) / U = C1 + C2 + C3 ,

т. е. при параллельном соединении конденсаторов общая емкость равна сумме емкостей отдельных конденсаторов.

Последовательное соединение конденсаторов

При последовательном соединении конденсаторов (рис. 3) на обкладках отдельных конденсаторов электрические заряды по величине равны: Q1 = Q2 = Q3 = Q

Действительно, от источника питания заряды поступают лишь на внешние обкладки цепи конденсаторов, а на соединенных между собой внутренних обкладках смежных конденсаторов происходит лишь перенос такого же по величине заряда с одной обкладки на другую (наблюдается электростатическая индукция), поэтому и на них по- являются равные и разноименые электрические заряды.

Рис. 3. Схема последовательного соединения конденсаторов

Напряжения между обкладками отдельных конденсаторов при их последовательном соединении зависят от емкостей отдельных конденсаторов: U1 = Q/C1 , U1 = Q/C 2, U1 = Q/C 3, а общее напряжение U = U1 + U2 + U3

Общая емкость равнозначного (эквивалентного) конденсатора C = Q / U = Q / ( U1 + U2 + U3 ), т. е. при последовательном соединении конденсаторов величина, обратная общей емкости, равна сумме обратных величин емкостей отдельных конденсаторов.

Формулы эквивалентных емкостей аналогичны формулам эквивалентных проводимостей.

Пример 1 . Три конденсатора, емкости которых C1 = 20 мкф, С2 = 25 мкф и С3 = 30 мкф, соединяются последовательно, необходимо определить общую емкость.

Общая емкость определяется из выражения 1/С = 1/С1 + 1/С2 + 1/С3 = 1/20 + 1/25 + 1/30 = 37/300, откуда С = 8,11 мкф.

Пример 2. 100 конденсаторов емкостью каждый 2 мкф соединены параллельно. Определить общую емкость. Общая емкость С = 100 Ск = 200 мкф.

Для достижения нужной емкости или при напряжении, превышающем номинальное напряжение, конденсаторы, могут соединяться последовательно или параллельно. Любое же сложное соединение состоит из нескольких комбинаций последовательного и параллельного соединений.

Последовательное соединение конденсаторов

При последовательном соединении, конденсаторы подключены таким образом, что только первый и последний конденсатор подключены к источнику ЭДС/тока одной из своих пластин. Заряд одинаков на всех пластинах, но внешние заряжаются от источника, а внутренние образуются только за счет разделения зарядов ранее нейтрализовавших друг друга. При этом заряд конденсаторов в батарее меньше, чем, если бы каждый конденсатор подключался бы отдельно. Следовательно, и общая емкость батареи конденсаторов меньше.

Напряжение на данном участке цепи соотносятся следующим образом:

Зная, что напряжение конденсатора можно представить через заряд и емкость, запишем:

Сократив выражение на Q, получим знакомую формулу:

Откуда эквивалентная емкость батареи конденсаторов соединенных последовательно:

Параллельное соединение конденсаторов

При параллельном соединении конденсаторов напряжение на обкладках одинаковое, а заряды разные.

Величина общего заряда полученного конденсаторами, равна сумме зарядов всех параллельно подключенных конденсаторов. В случае батареи из двух конденсаторов:

Так как заряд конденсатора

А напряжения на каждом из конденсаторов равны, получаем следующее выражение для эквивалентной емкости двух параллельно соединенных конденсаторов

Пример 1

Какова результирующая емкость 4 конденсаторов включенных последовательно и параллельно, если известно что С1 = 10 мкФ, C2 = 2 мкФ, C3 = 5 мкФ, а C4 = 1 мкФ?

При последовательном соединении общая емкость равна:

При параллельном соединении общая емкость равна:

Пример 2

Определить результирующую емкость группы конденсаторов подключенных последовательно-параллельно, если известно, что С1 = 7 мкФ, С2 = 2 мкФ, С3 = 1 мкФ.

Сначала найдем общую емкость параллельного участка цепи:

Затем найдем общую емкость для всей цепи:

По сути, расчет общей емкости конденсаторов схож с расчетом общего сопротивления цепи в случае с последовательным или параллельным соединением, но при этом, зеркально противоположен.

Статьи, Схемы, Справочники

Для получения нужной емкости или при напряжении сети, превышающем номинальное напряжение конденсатора, они соединяются последовательно, параллельно или смешанно. При последовательном соединении рис. Последовательное соединение конденсаторов. Параллельное соединение конденсаторов.

Поиск данных по Вашему запросу:

Дождитесь окончания поиска во всех базах.

По завершению появится ссылка для доступа к найденным материалам.
Перейти к результатам поиска >>>

ПОСМОТРИТЕ ВИДЕО ПО ТЕМЕ: Физика. Решение задач. Конденсаторы. Выполнялка 24

Параллельное и последовательное соединение конденсаторов

Не всегда удаётся подобрать конденсатор нужного номинала. Очень часто начинающие домашние мастера, обнаружив поломку прибора, стараются самостоятельно обнаружить причину. Увидев сгоревшую деталь, они стараются найти подобную, а если это не удаётся, несут прибор в ремонт. На самом деле, не обязательно, чтобы показатели совпадали. Можно использовать конденсаторы меньшего номинала, соединив их в цепь. Главное — сделать это правильно. При этом достигается сразу 3 цели — поломка устранена, приобретён опыт, сэкономлены средства семейного бюджета.

Попробуем разобраться, какие способы соединения существуют и на какие задачи рассчитаны последовательное и параллельное соединение конденсаторов. На самом деле всё довольно просто.

При параллельном соединении расчёт общей ёмкости можно вычислить путём простейшего сложения всех конденсаторов. Получается, что подобный монтаж подразумевает подключение всех пластин конденсаторов к точкам питания. Такой способ встречается наиболее часто. Но может произойти ситуация, когда важно увеличить напряжение. Разберёмся, каким образом это сделать. При использовании способа последовательного подключения конденсаторов напряжение в цепи возрастает. Рассмотрим изменения ёмкости и напряжения при последовательном включении на примере.

Дано: 3 конденсатора с напряжением В и ёмкостью мкф. Подключив их последовательно, получим:. Выполняют такое соединение в том случае, если есть опасность пробоя диэлектрика конденсатора при подаче напряжения в цепь. Полезно знать!

Применяют также последовательное и параллельное соединение резисторов и конденсаторов. Это делается с целью снижения подаваемого на конденсатор напряжения и исключения его пробоя. Однако следует учитывать, что напряжения должно быть достаточно для работы самого прибора. Такое подключение его ещё называют последовательно-параллельным применяют в случае необходимости увеличения, как ёмкости, так и напряжения. Здесь вычисление общих параметров немного сложнее, но не настолько, чтобы нельзя было разобраться начинающему радиолюбителю.

Для начала посмотрим, как выглядит такая схема. Смешанное соединение конденсаторов решает задачи, которые не под силу параллельным и последовательным схемам. Его можно использовать при подключении электродвигателей либо иного оборудования, его монтаж возможен отдельными участками.

Монтаж его намного проще за счёт возможности выполнения отдельными частями. Интересно знать! Многие радиолюбители считают этот способ более простым и приемлемым, чем два предыдущих. На самом деле, так и есть, если полностью понять алгоритм действий и научиться пользоваться им правильно. Соединяя конденсаторы, в особенности электролитические, обратите внимание на строгое соблюдение полярности.

Все элементы должны быть однотипны —плёночные, керамические, слюдяные либо металлобумажные. Выход из строя конденсаторов часто происходит по вине производителя, экономящего на деталях чаще это приборы китайского производства. Поэтому правильно рассчитанные и собранные в схему элементы будут работать намного дольше. Конечно, при условии отсутствия замыкания в цепи, при котором работа конденсаторов невозможна в принципе. А что делать, если необходимая ёмкость неизвестна?

Не каждому хочется самостоятельно рассчитывать необходимую ёмкость конденсаторов вручную, а у кого-то на это просто нет времени. Для удобства производства подобных действий редакция Seti.

В работе он необычайно прост. Программы, в которые заложены все алгоритмы и формулы последовательного соединения конденсаторов, а также вычислений необходимой ёмкости, моментально выдаст необходимый результат.

Первое, что для этого необходимо сделать — рассчитать, с какой силой притягиваются обкладки друг к другу. Однако это не окончательная формула, которая нам необходима. Конечно, для начинающего радиолюбителя все эти расчёты могут показаться сложными и непонятными, но при желании и некоторой усидчивости с ними можно разобраться.

Вникнув в смысл, он поразится, насколько просто производятся все эти расчёты. По сути, расчёт энергии применяется редко, однако есть области, в которых это знать необходимо. К примеру, фотовспышка камеры — здесь вычисление показателя энергии очень важно. Она накапливается за определённое время несколько секунд , а вот выдаётся мгновенно. Получается, что конденсатор сравним с аккумулятором — разница лишь в ёмкости. Порой без соединения конденсаторов не обойтись, ведь не всегда можно подобрать подходящие по номиналам.

Поэтому знание того как это сделать может выручить при поломке бытовой техники или электроники, что позволит значительно сэкономить на оплате труда специалиста по ремонту.

Как наверняка уже понял Уважаемый читатель, сделать это несложно и под силу даже начинающим домашним мастерам. А значит стоит потратить немного своего драгоценного времени и разобраться в алгоритме действий и правилах их выполнения. Надеемся, что информация, изложенная в сегодняшней статье, была полезна нашим читателям.

Возможно, у Вас остались какие-либо вопросы? В этом случае их можно изложить в обсуждении ниже. Редакция Seti. Если же Вы имеете опыт самостоятельного соединения конденсаторов неважно, положительный он или отрицательный , убедительная просьба поделиться им с другими читателями. Это поможет начинающим мастерам более полно понять алгоритм действий и избежать ошибок. Пишите, делитесь, спрашивайте. А напоследок мы предлагаем посмотреть короткий, но довольно информативный видеоролик по сегодняшней теме.

Отдельные конденсаторы могут быть соединены друг с другом различным образом. При этом во всех случаях можно найти емкость некоторого равнозначного конденсатора, который может заменить ряд соединенных между собой конденсаторов. Для равнозначного конденсатора выполняется условие: если подводимое к обкладкам равнозначного конденсатора напряжение равно напряжению, подводимому к крайним зажимам группы конденсаторов, то равнозначный конденсатор накопит такой же заряд, как и группа конденсаторов.

На рис. При последовательном соединении конденсаторов рис. Действительно, от источника питания заряды поступают лишь на внешние обкладки цепи конденсаторов, а на соединенных между собой внутренних обкладках смежных конденсаторов происходит лишь перенос такого же по величине заряда с одной обкладки на другую наблюдается электростатическая индукция , поэтому и на них по- являются равные и разноименые электрические заряды. Пример 1. Пример 2. Определить общую емкость. В электронных и радиотехнических схемах широкое распространение получило параллельное и последовательное соединение конденсаторов.

В первом случае соединение осуществляется без каких-либо общих узлов, а во втором варианте все элементы объединяются в два узла и не связаны с другими узлами, если это заранее не предусмотрено схемой. При последовательном соединении два и более конденсаторов соединяются в общую цепь таким образом, что каждый предыдущий конденсатор соединяется с последующим лишь в одной общей точке.

Ток i , осуществляющий зарядку последовательной цепи конденсаторов будет иметь одинаковое значение для каждого элемента, поскольку он проходит только по единственно возможному пути. В связи с одинаковым значением тока, протекающего через конденсаторы с последовательным соединением, величина заряда, накопленного каждым из них, будет одинаковой, независимо от емкости.

Такое становится возможным, поскольку заряд, приходящий с обкладки предыдущего конденсатора, накапливается на обкладке последующего элемента цепи. Если рассмотреть три конденсатора С1. С2 и С3. В конечном итоге величина эффективной площади обкладок будет уменьшена до площади обкладок конденсатора с самыми минимальными размерами. Полное заполнение обкладок электрическим зарядом, делает невозможным дальнейшее прохождение по нему тока. В результате, движение тока прекращается во всей цепи, соответственно прекращается и зарядка всех остальных конденсаторов.

Общее расстояние между обкладками при последовательном соединении представляет собой сумму расстояний между обкладками каждого элемента.

В результате соединения в последовательную цепь, формируется единый большой конденсатор, площадь обкладок которого соответствует обкладкам элемента с минимальной емкостью. Падение напряжения на каждый конденсатор будет разным, в зависимости от емкости. Таким образом, с уменьшением емкости конденсатора на него падает более высокое напряжение. Главная особенность такой схемы заключается в прохождении электрической энергии только в одном направлении.

Поэтому в каждом конденсаторе значение тока будет одинаковым. Каждый накопитель в последовательной цепи накапливает равное количество энергии, независимо от емкости. То есть емкость может воспроизводиться за счет энергии, присутствующей в соседнем накопителе. Параллельным считается такое соединение, при котором конденсаторы соединяются между собой двумя контактами. Таким образом в одной точке может соединяться сразу несколько элементов.

Данный вид соединения позволяет сформировать единый конденсатор с большими размерами, площадь обкладок которого будет равна сумме площадей обкладок каждого, отдельно взятого конденсатора. В связи с тем, что емкость конденсаторов находится в прямой пропорциональной зависимости с площадью обкладок, общая емкость составить суммарное количество всех емкостей конденсаторов, соединенных параллельно.

Поскольку разность потенциалов возникает лишь в двух точках, то на все конденсаторы, соединенные параллельно, будет падать одинаковое напряжение.

Сила тока в каждом из них будет отличаться, в зависимости от емкости и значения напряжения. Таким образом, последовательное и параллельное соединение, применяемое в различных схемах, позволяет выполнять регулировку различных параметров на тех или иных участках. Казалось бы, зачем это надо, ведь если на принципиальной схеме указано, что в данном месте схемы должен быть установлен конденсатор на 47 микрофарад, значит, берём и ставим.

Но, согласитесь, что в мастерской даже заядлого электронщика может не оказаться конденсатора с необходимым номиналом!

Конденсатор последовательное соединение

В электрических цепях применяются различные способы соединения конденсаторов. Соединение конденсаторов может производиться: последовательно , параллельно и последовательно-параллельно последнее иногда называют смешанное соединение конденсаторов. Существующие виды соединения конденсаторов показаны на рисунке 1. Тогда общая емкость конденсаторов при параллельном соединении равна сумме емкостей всех соединенных конденсаторов. Тогда для параллельного соединения конденсаторов будет справедлива следующая формула:. Напряжения на различных конденсаторах будут, вообще говоря, различными, так как для заряда одним и тем же количеством электричества конденсаторов различной емкости всегда требуются различные напряжения.

Последовательное соединение конденсаторов

Последовательное соединение конденсаторов. Логическая зарядка конденсаторов происходит как показано на рис. Приходя из цепи, электрон останавливается на левой обкладке пластине конденсатора. При этом, благодаря своему электрическому полю электризация через влияние , он выбивает другой электрон с правой обкладки, уходящий дальше в цепь рис. Этот образовавшийся электрон приходит на левую обкладку следующего конденсатора, соединённого последовательно. И всё повторяется снова. Кроме того, напряжение на последовательно соединённой батареи конденсаторов есть сумма напряжений на каждом из элементов аналог последовательного сопротивления проводников. Учитывая определение электроёмкости :.

Последовательное и параллельное соединение конденсаторов

Соединения конденсаторов. Параллельное соединение конденсаторов. Обкладки конденсаторов соединяют попарно, то есть в системе остается два изолированных проводника, которые и представляют собой обкладки нового конденсатора. Последовательное соединение конденсаторов.

При последовательном соединении конденсаторов их суммарная емкость — советы электрика

Параллельное соединение конденсаторов. Конденсаторы соединяют параллельно для увеличения общей емкости батареи. При таком соединении рис. Таким образом, полная емкость батареи параллельно соединенных конденсаторов равны сумме емкостей отдельных конденсаторов. Последовательное соединение конденсаторов. Последовательное соединение конденсаторов применяется в тех случаях, когда изоляция диэлектрик одного конденсатора не может выдержать рабочее напряжение установки.

Последовательное соединение конденсаторов для подбора емкости. Соединение конденсаторов

Последовательное соединение конденсаторов — батарея, образованная цепочкой конденсаторов. Отсутствует ветвление, выход одного элемента подключается к входу следующего. При последовательном соединении конденсаторов заряд каждого равноценен. Обусловлено природным принципом равновесия. Используя равенство, находим:.

СОЕДИНЕНИЕ КОНДЕНСАТОРОВ

Код для вставки без рекламы с прямой ссылкой на сайт. Код для вставки с рекламой без прямой ссылки на сайт. Скопируйте и вставьте этот код на свою страничку в то место, где хотите, чтобы отобразился калькулятор.

Схемы в электротехнике состоят из электрических элементов, в которых способы соединения конденсаторов могут быть разными. Надо понимать, как правильно подключить конденсатор. Отдельные участки цепи с подключенными конденсаторами можно заменить одним эквивалентным элементом. Он заменит ряд конденсаторов, но должно выполняться обязательное условие: когда напряжение, подводимое к обкладкам эквивалентного конденсатора, равняется напряжению на входе и выходе группы заменяющихся конденсаторов, тогда заряд емкости будет такой же, как и на группе емкостей. Для понимания вопроса, как подключить конденсатор в любой схеме, рассмотрим виды его включения. Параллельное соединение конденсаторов — это когда все пластины подключаются к точкам включения цепи, образовывая батарею емкостей.

Отдельные конденсаторы могут быть соединены друг с другом различным образом. При этом во всех случаях можно найти емкость некоторого равнозначного конденсатора, который может заменить ряд соединенных между собой конденсаторов. Для равнозначного конденсатора выполняется условие: если подводимое к обкладкам равнозначного конденсатора напряжение равно напряжению, подводимому к крайним зажимам группы конденсаторов, то равнозначный конденсатор накопит такой же заряд, как и группа конденсаторов. На рис. При последовательном соединении конденсаторов рис.

Изучите соединения конденсаторов — последовательные и параллельные. Как выглядят последовательно и параллельно соединенные конденсаторы, емкость, схемы. Посмотрим на схему, где конденсаторы соединены последовательно. Обратная общая емкость достигает суммы обратных значений емкости каждого отдельного конденсатора:.

НАШ САЙТ РЕКОМЕНДУЕТ:

Метки:  

8.3: Последовательные и параллельные конденсаторы

Несколько конденсаторов могут быть соединены вместе для использования в различных приложениях. Несколько подключений конденсаторов ведут себя как один эквивалентный конденсатор. Общая емкость этого эквивалентного одиночного конденсатора зависит как от отдельных конденсаторов, так и от способа их подключения. Конденсаторы могут быть организованы в два простых и распространенных типа соединений, известных как серии и параллельно , для которых мы можем легко вычислить общую емкость.Эти две основные комбинации, последовательная и параллельная, также могут использоваться как часть более сложных соединений.

Серия конденсаторов

На рисунке \ (\ PageIndex {1} \) показано последовательное сочетание трех конденсаторов, расположенных в ряд внутри схемы. Как и в случае любого конденсатора, емкость комбинации связана как с зарядом, так и с напряжением:

\ [C = \ dfrac {Q} {V}. \]

Когда эта последовательная комбинация подключена к батарее с напряжением В , каждый из конденсаторов получает идентичный заряд Q .Чтобы объяснить, сначала обратите внимание, что заряд на пластине, подключенной к положительной клемме батареи, равен \ (+ Q \), а заряд на пластине, подключенной к отрицательной клемме, равен \ (- Q \). Затем на других пластинах индуцируются заряды, так что сумма зарядов на всех пластинах и сумма зарядов на любой паре пластин конденсатора равна нулю. Однако падение потенциала \ (V_1 = Q / C_1 \) на одном конденсаторе может отличаться от падения потенциала \ (V_2 = Q / C_2 \) на другом конденсаторе, потому что, как правило, конденсаторы могут иметь разные емкости.Последовательная комбинация двух или трех конденсаторов напоминает один конденсатор с меньшей емкостью. Как правило, любое количество последовательно соединенных конденсаторов эквивалентно одному конденсатору, емкость которого (называемая эквивалентной емкостью ) меньше наименьшей из емкостей в последовательной комбинации. Заряд этого эквивалентного конденсатора такой же, как заряд любого конденсатора в последовательной комбинации: то есть , все конденсаторы последовательной комбинации имеют одинаковый заряд .Это происходит из-за сохранения заряда в цепи. Когда заряд Q в последовательной цепи удаляется с пластины первого конденсатора (который мы обозначаем как \ (- Q \)), он должен быть помещен на пластину второго конденсатора (который мы обозначаем как \ ( + Q \)) и т. Д.

Рисунок \ (\ PageIndex {1} \): (a) Три конденсатора соединены последовательно. Величина заряда на каждой пластине равна Q. (b) Сеть конденсаторов на (a) эквивалентна одному конденсатору, который имеет меньшую емкость, чем любая из отдельных емкостей на (a), а заряд на его пластинах равен В.

Мы можем найти выражение для полной (эквивалентной) емкости, рассматривая напряжения на отдельных конденсаторах. Потенциалы на конденсаторах 1, 2 и 3 равны, соответственно, \ (V_1 = Q / C_1 \), \ (V_2 = Q / C_2 \) и \ (V_3 = Q / C_3 \). Эти потенциалы должны суммироваться с напряжением батареи, давая следующий баланс потенциалов:

\ [V = V_1 + V_2 + V_3. \]

Потенциал \ (V \) измеряется на эквивалентном конденсаторе, который держит заряд \ (Q \) и имеет эквивалентную емкость \ (C_S \).Вводя выражения для \ (V_1 \), \ (V_2 \) и \ (V_3 \), получаем

\ [\ dfrac {Q} {C_S} = \ dfrac {Q} {C_1} + \ dfrac {Q} {C_2} + \ dfrac {Q} {C_3}. \]

Отменяя заряд Q , мы получаем выражение, содержащее эквивалентную емкость \ (C_S \) трех последовательно соединенных конденсаторов:

\ [\ dfrac {1} {C_S} = \ dfrac {1} {C_1} + \ dfrac {1} {C_2} + \ dfrac {1} {C_3}. \]

Это выражение можно обобщить на любое количество конденсаторов в последовательной сети.

Комбинация серии

Для конденсаторов, соединенных последовательно, эквивалентная емкость, обратная величине, является суммой обратных величин индивидуальных емкостей:

\ [\ dfrac {1} {C_S} = \ dfrac {1} {C_1} + \ dfrac {1} {C_2} + \ dfrac {1} {C_3} + \ dots \ label {capseries} \]

Пример \ (\ PageIndex {1} \): эквивалентная емкость последовательной сети

Найдите общую емкость для трех последовательно соединенных конденсаторов, учитывая, что их отдельные емкости равны \ (1.000 мкФ \), \ (5.000 мкФ \) и \ (8.000 мкФ \).

Стратегия

Поскольку в этой сети всего три конденсатора, мы можем найти эквивалентную емкость, используя уравнение \ ref {capseries} с тремя членами.

Решение

Вводим указанные емкости в уравнение \ ref {capseries}:

\ [\ begin {align *} \ dfrac {1} {C_S} & = \ dfrac {1} {C_1} + \ dfrac {1} {C_2} + \ dfrac {1} {C_3} \\ [4pt] & = \ dfrac {1} {1.000 \ mu F} + \ dfrac {1} {5.000 \ mu F} + \ dfrac {1} {8.000 \ mu F} \\ [4pt] & = \ dfrac {1.325} {\ mu F}. \ End {align *} \]

Теперь инвертируем этот результат и получаем

\ [\ begin {align *} C_S & = \ dfrac {\ mu F} {1.325} \\ [4pt] & = 0.755 \ mu F. \ end {align *} \ nonumber \]

Значение

Обратите внимание, что в последовательной сети конденсаторов эквивалентная емкость всегда меньше наименьшей отдельной емкости в сети.

Параллельная комбинация конденсаторов

Параллельная комбинация трех конденсаторов, одна пластина каждого конденсатора подключена к одной стороне цепи, а другая пластина подключена к другой стороне, показана на рисунке \ (\ PageIndex {2a} \).Поскольку конденсаторы соединены параллельно, все они имеют одинаковое напряжение V на пластинах . Однако каждый конденсатор в параллельной сети может накапливать свой заряд. Чтобы найти эквивалентную емкость \ (C_p \) параллельной сети, отметим, что общий заряд Q , хранящийся в сети, является суммой всех отдельных зарядов:

\ [Q = Q_1 + Q_2 + Q_3. \]

В левой части этого уравнения используется соотношение \ (Q = C_pV \), которое выполняется для всей сети.В правой части уравнения мы используем соотношения \ (Q_1 = C_1 V \), \ (Q_2 = C_2V \) и \ (Q_3 = C_3V \) для трех конденсаторов в сети. Таким образом получаем

\ [C_pV = C_1V + C_2V + C_3V. \]

Это уравнение в упрощенном виде представляет собой выражение для эквивалентной емкости параллельной сети из трех конденсаторов:

\ [C_p = C_1 + C_2 + C_3. \]

Это выражение легко обобщается на любое количество конденсаторов, включенных параллельно в сеть.

Параллельная комбинация

Для конденсаторов, соединенных параллельно, эквивалентная (полезная) емкость представляет собой сумму всех индивидуальных емкостей в сети,

\ [C_p = C_1 + C_2 + C_3 + … \ label {capparallel} \]

Рисунок \ (\ PageIndex {2} \): (a) Три конденсатора подключены параллельно. Каждый конденсатор подключен непосредственно к батарее. (b) Заряд эквивалентного конденсатора представляет собой сумму зарядов отдельных конденсаторов.

Пример \ (\ PageIndex {2} \): эквивалентная емкость параллельной сети

Найдите полезную емкость для трех конденсаторов, соединенных параллельно, учитывая, что их индивидуальные емкости равны \ (1.0 \ mu F \), \ (5.0 \ mu F \) и \ (8.0 \ mu F \).

Стратегия

Поскольку в этой сети всего три конденсатора, мы можем найти эквивалентную емкость, используя уравнение \ ref {capparallel} с тремя членами.

Решение

Ввод заданных емкостей в уравнение \ ref {capparallel} дает

\ [\ begin {align *} C_p & = C_1 + C_2 + C_3 \\ [4pt] & = 1.0 \ mu F + 5.0 \ mu F + 8.0 \ mu F \\ [4pt] & = 14.0 \ mu F.\ end {align *} \]

Значение

Обратите внимание, что в параллельной сети конденсаторов эквивалентная емкость всегда больше, чем любая из отдельных емкостей в сети.

Конденсаторные сети обычно представляют собой комбинацию последовательных и параллельных соединений, как показано на Рисунке \ (\ PageIndex {3} \). Чтобы найти чистую емкость таких комбинаций, мы определяем части, которые содержат только последовательные или только параллельные соединения, и находим их эквивалентные емкости.Мы повторяем этот процесс, пока не сможем определить эквивалентную емкость всей сети. Следующий пример иллюстрирует этот процесс.

Рисунок \ (\ PageIndex {3} \): (a) Эта схема содержит как последовательные, так и параллельные соединения конденсаторов. (b) \ (C_1 \) и \ (C_2 \) идут последовательно; их эквивалентная емкость \ (C_S \) c) Эквивалентная емкость \ (C_S \) подключена параллельно с \ (C_3 \). Таким образом, эквивалентная емкость всей сети является суммой \ (C_S \) и \ (C_3 \).

Пример \ (\ PageIndex {3} \): эквивалентная емкость сети

Найдите общую емкость комбинации конденсаторов, показанной на рисунке \ (\ PageIndex {3} \). Предположим, что емкости известны с точностью до трех десятичных знаков (\ (C_1 = 1.000 мкФ, C_2 = 5.000 мкФ, C_3 = 8.000 мкФ \)). Округлите ответ до трех десятичных знаков.

Стратегия

Сначала мы определяем, какие конденсаторы включены последовательно, а какие — параллельно. Конденсаторы \ (C_1 \) и \ (C_2 \) включены последовательно.Их комбинация, обозначенная \ (C_S \), параллельна \ (C_3 \).

Решение

Поскольку \ (C_1 \) и \ (C_2 \) включены последовательно, их эквивалентная емкость \ (C_S \) получается с помощью уравнения \ ref {capseries}:

\ [\ begin {align *} \ dfrac {1} {C_S} & = \ dfrac {1} {C_1} + \ dfrac {1} {C_2} \\ [4pt] & = \ dfrac {1} {1.000 \ mu F} + \ dfrac {1} {5.000 \ mu F} \\ [4pt] & = \ dfrac {1.200} {\ mu F} \ end {align *} \]

Для этого

\ [C_S = 0,833 \ mu F. \ nonumber \]

Емкость

\ (C_S \) соединена параллельно с третьей емкостью \ (C_3 \), поэтому мы используем уравнение \ ref {capparallel}, чтобы найти эквивалентную емкость C всей сети:

\ [\ begin {align *} C & = C_S + C_3 \\ [4pt] & = 0.833 \ mu F + 8.000 \ mu F \\ [4pt] & = 8.833 \ mu F. \ end {align *} \]

Сеть конденсаторов

Определите чистую емкость C комбинации конденсаторов, показанной на рисунке \ (\ PageIndex {4} \), когда емкости равны \ (C_1 = 12,0 мкФ, C_2 = 2,0 мкФ \) и \ (C_3 = 4,0 мкФ \). Когда в комбинации сохраняется разность потенциалов 12,0 В, найдите заряд и напряжение на каждом конденсаторе.

Рисунок \ (\ PageIndex {4} \): (a) Комбинация конденсаторов.(b) Эквивалентная комбинация из двух конденсаторов. Стратегия

Сначала мы вычисляем чистую емкость \ (C_ {23} \) параллельного соединения \ (C_2 \) и \ (C_3 \). Тогда C — это чистая емкость последовательного соединения \ (C_1 \) и \ (C_ {23} \). Мы используем соотношение \ (C = Q / V \), чтобы найти заряды \ (Q_1, Q_2 \) и \ (Q_3 \), а также напряжения \ (V_1, V_2 \) и \ (V_3 \) на конденсаторы 1, 2 и 3 соответственно.

Решение Эквивалентная емкость для \ (C_2 \) и \ (C_3 \) составляет

\ [C_ {23} = C_2 + C_3 = 2.0 мк F + 4,0 мк F = 6,0 мк F. \]

Вся комбинация из трех конденсаторов эквивалентна двум последовательно включенным конденсаторам,

\ [\ dfrac {1} {C} = \ dfrac {1} {12.0 \ mu F} + \ dfrac {1} {6.0 \ mu F} = \ dfrac {1} {4.0 \ mu F} \ Rightarrow C = 4,0 мкм F. \]

Рассмотрим эквивалентную комбинацию из двух конденсаторов на рисунке \ (\ PageIndex {2b} \). Поскольку конденсаторы включены последовательно, они имеют одинаковый заряд, \ (Q_1 = Q_ {23} \). Кроме того, конденсаторы разделяют разность потенциалов 12,0 В, поэтому

\ [12.0 V = V_1 + V_ {23} = \ dfrac {Q_1} {C_1} + \ dfrac {Q_ {23}} {C_ {23}} = \ dfrac {Q_1} {12.0 \ mu F} + \ dfrac {Q_1 } {6.0 \ mu F} \ Rightarrow Q_1 = 48.0 \ mu C. \]

Теперь разность потенциалов на конденсаторе 1 равна

.

\ [V_1 = \ dfrac {Q_1} {C_1} = \ dfrac {48.0 \ mu C} {12.0 \ mu F} = 4.0 V. \]

Поскольку конденсаторы 2 и 3 подключены параллельно, они имеют одинаковую разность потенциалов:

\ [V_2 = V_3 = 12,0 В — 4,0 В = 8,0 В. \]

Следовательно, заряды на этих двух конденсаторах равны, соответственно,

\ [Q_2 = C_2V_2 = (2.0 мкФ) (8,0 В) = 16,0 мкФ, \]

\ [Q_3 = C_3V_3 = (4,0 мкФ) (8,0 В) = 32,0 мкФ \]

Значимость Как и ожидалось, чистая плата за параллельную комбинацию \ (C_2 \) и \ (C_3 \) составляет \ (Q_ {23} = Q_2 + Q_3 = 48,0 \ mu C. \)

Упражнение \ (\ PageIndex {1} \)

Определите чистую емкость C каждой сети конденсаторов, показанной ниже. Предположим, что \ (C_1 = 1,0 пФ, C_2 = 2,0 пФ, C_3 = 4,0 пФ \) и \ (C_4 = 5,0 пФ \). Найдите заряд на каждом конденсаторе, предполагая, что разность потенциалов равна 12.0 В в каждой сети.

Ответьте на

\ (C = 0,86 пФ, Q_1 = 10 пКл, Q_2 = 3,4 пКл, Q_3 = 6,8 пКл \)

Ответ b

\ (C = 2,3 пФ, Q_1 = 12 пКл, Q_2 = Q_3 = 16 пКл \)

Ответ c

\ (C = 2,3 пФ, Q_1 = 9,0 пКл, Q_2 = 18 пКл, Q_3 = 12 пКл, Q_4 = 15 пКл \)

Цепи конденсаторов серии

и параллельные

Разница между Кулоном и Фарадом

Раньше переходя к последовательным и параллельным цепям конденсаторов, сначала посмотрите на разница между кулоном и фарадом, потому что многие люди запутаться в определении разницы между кулоном и Фарад.

Электрический заряд измеряется в кулонах. Один кулон (1С) равен равно количеству заряда, передаваемого за одну секунду Текущий одного Ампера (1А).

Емкость является способность тела или устройства накапливать электрический заряд. Емкость измеряется в фарадах (Ф). Устройство с большим Емкость (96F) сохранит большой заряд.Точно так же устройство с малой емкостью (1F) будет хранить небольшая сумма заряда.

серии конденсаторная схема

А последовательный конденсатор схема — это электронная схема, в которой все конденсаторы подключаются друг за другом по одному и тому же пути, поэтому что к каждому конденсатору протекает одинаковый заряд или ток.

общая емкость цепи последовательного конденсатора получается как сложение обратных величин (1 / C) значений емкости отдельных конденсаторов, а затем взяв обратную величину Общая.

Для Например, если три конденсатора соединены последовательно. Тогда общая емкость цепи


Все ток или заряд, протекающий через первый конденсатор, другого пути нет.Следовательно, он также должен проходить через второй конденсатор, третий конденсатор, четвертый конденсатор и т. д. на.

Пример:

А Схема последовательного конденсатора показана на рисунке ниже. В схема состоит из трех конденсаторов, которые включены в последовательный и источник постоянного напряжения.

емкости из трех конденсаторов: C 1 = 2F, C 2 = 4F, C 3 = 6F и постоянное напряжение = 10 В.

Как как показано на рисунке, положительный полюс батареи постоянного тока подключается к правой боковой пластине конденсатора С 3 отрицательная клемма батареи постоянного тока подключена к левая боковая пластина конденсатора С 1 .

Когда а напряжение приложено к цепи, отрицательные заряды в правой боковой пластине конденсатора С 3 находятся притянул к плюсовой клемме аккума.Это вызывает Недостаток отрицательных зарядов в правой боковой пластине C 3 . В итоге правая боковая пластина конденсатора С 3 заряжен положительно.

Аналогично, в положительные заряды в левой боковой пластине конденсатора С 1 притягиваются к отрицательной клемме аккумулятора. Этот вызывает нехватку положительных зарядов в левой боковой пластине из C 1 .В результате левая боковая пластина Конденсатор С 1 заряжен отрицательно.

отрицательные заряды в левой боковой пластине конденсатора С 1 отталкивать отрицательные заряды в правой боковой пластине конденсатор С 1 . Это вызывает отрицательные заряды. сток с правой боковой пластины конденсатора С 1 к левой боковой пластине конденсатора С 2 .Как В результате правая боковая пластина конденсатора С 1 оказывается положительно заряжена и левая боковая пластина конденсатора С 2 заряжен отрицательно.

отрицательные заряды в левой боковой пластине конденсатора С 2 отталкивать отрицательные заряды в правой боковой пластине конденсатор С 2 .Это вызывает отрицательные заряды. сток с правой боковой пластины конденсатора С 2 к левой боковой пластине конденсатора С 3 . Как В результате правая боковая пластина конденсатора С 2 оказывается положительно заряжена и левая боковая пластина конденсатора С 3 заряжен отрицательно.

Таким образом, все три конденсатора заряжаются.

ср знайте, что ток означает поток заряда. С того же ток течет через все три конденсатора, поэтому каждый конденсатор будет держать такой же заряд. Это означает, что если один конденсатор держит заряд 2C, тогда остальные конденсаторы тоже держит такой же заряд 2С.

Так если вы обнаружите заряд на одном из конденсаторов, у вас нашел заряд на всех оставшихся конденсаторах.

В чтобы найти заряд на каждом конденсаторе, сначала нам нужно найти общую емкость или эквивалентную емкость.

общая емкость эквивалентного конденсатора


Автор используя формулу C = Q / V, легко найти заряд хранится на эквивалентном конденсаторе.


Начисление на каждого физ. конденсаторы, подключенные последовательно, такие же, как заряд на эквивалентном конденсатор.

Итак, так как заряд на эквивалент конденсатор был 10,91 кулонов, заряд на каждой из отдельные конденсаторы, включенные последовательно, будут иметь 10,91 кулонов.

Следовательно,

Плата за C 1 = 10.91 C

Заряд на C 2 = 10.91 C

Заряд на C 3 = 10.91 С

Однако в цепи последовательного конденсатора напряжение на каждом индивидуальный конденсатор разный.

ср легко найти напряжение на каждом отдельном конденсаторе по формуле C = Q / V

емкость и заряд на каждом отдельном конденсаторе известны. Итак, мы нужно найти неизвестное напряжение.

В = Q / C

напряжение на конденсаторе (C 1 ) составляет В 1 = Q / C 1 = 10,91 / 2 = 5,455 В

напряжение на конденсаторе (C 2 ) составляет В 2 = Q / C 2 = 10,91 / 4 = 2,727 В

напряжение на конденсаторе (C 3 ) составляет V 3 = Q / С 3 = 10.91/6 = 1,818 В

полное напряжение в цепи последовательного конденсатора равно сумма всех отдельных напряжений, сложенных вместе.

Т.е. V = V 1 + V 2 + V 3 = 5,455 + 2,727 + 1,818 = 10 В

Параллельный конденсаторная схема

А параллельная конденсаторная схема — это электронная схема, в которой все конденсаторы соединены бок о бок в разных пути, чтобы тот же заряд или ток не проходили через каждый конденсатор.

Когда на параллельную цепь подается напряжение, каждый конденсатор получит другой заряд. Конденсатор с высоким емкость получит больший заряд, тогда как конденсатор с чем меньше емкость, тем меньше будет заряда. Например, восьмерка Фарадный конденсатор (8F) получит больше заряда, чем четыре фарада конденсатор (4Ф) попадает.

Путь конденсаторы параллельно будет увеличиваться размер пластин конденсатора без увеличения расстояния между ними. Итак, общая емкость параллельной конденсаторной цепи получается просто суммируя значения емкости отдельных конденсаторы.

Пример:

А Схема параллельного конденсатора показана на рисунке ниже.В схема состоит из трех конденсаторов, которые включены в параллельный и источник постоянного напряжения.

Если Значения трех конденсаторов: C 1 = 8F, C 2 = 4F, C 3 = 2F и батарея постоянного тока = 10 В, тогда

общая емкость составляет C T = C 1 + C 2 + C 3 = 8 + 4 + 2 = 14F

В принципиальная схема, нижние обкладки трех конденсаторов напрямую подключены к положительной клемме аккумулятора а верхние обкладки трех конденсаторов непосредственно подключен к отрицательной клемме аккумуляторной батареи.Следовательно, напряжение на всех трех конденсаторах одинаковое, что составляет равно напряжению АКБ постоянного тока (10 В).

Однако в параллельной цепи конденсаторов заряд сохраняется на каждом конденсатор будет другим.

Автор используя формулу емкости, легко найти заряд хранится на каждом конденсаторе.

И.е. C = Q / V

Q = C × V

заряд, накопленный на конденсаторе (C 1 ), составляет Q 1 = С 1 × V = 8 × 10 = 80 С

заряд, накопленный на конденсаторе (C 2 ), составляет Q 2 = С 2 × V = 4 × 10 = 40 С

Заряд, накопленный на конденсаторе (C 3 ), составляет Q 3 = C 3 × V = 2 × 10 = 20 С

Общий заряд, хранящийся в параллельном конденсаторная цепь равна сумме всех отдельных заряды конденсатора складываются.

Т.е. Q T = Q 1 + Q 2 + Q 3 = 80 + 40 + 20 = 140 C


19.6 Последовательные и параллельные конденсаторы — College Physics chapters 1-17

Сводка

  • Выведите выражения для полной емкости последовательно и параллельно.
  • Обозначает последовательные и параллельные части в комбинации подключения конденсаторов.
  • Рассчитайте эффективную емкость последовательно и параллельно с учетом индивидуальных емкостей.

Несколько конденсаторов могут быть соединены вместе в различных приложениях. Несколько подключений конденсаторов действуют как один эквивалентный конденсатор. Общая емкость этого эквивалентного одиночного конденсатора зависит как от отдельных конденсаторов, так и от способа их подключения.Существует два простых и распространенных типа соединений, называемых серией и параллельными , для которых мы можем легко вычислить общую емкость. Некоторые более сложные соединения также могут быть связаны с комбинациями последовательного и параллельного.

На рисунке 1 (а) показано последовательное соединение трех конденсаторов с приложенным напряжением. Как и для любого конденсатора, емкость комбинации связана с зарядом и напряжением [латекс] \ boldsymbol {C = \ frac {Q} {V}} [/ latex].

Обратите внимание на рис. 1, что противоположные заряды величиной [латекс] \ boldsymbol {Q} [/ latex] текут по обе стороны от первоначально незаряженной комбинации конденсаторов при приложении напряжения [латекс] \ boldsymbol {V} [/ латекс] . Для сохранения заряда необходимо, чтобы на пластинах отдельных конденсаторов создавались заряды одинаковой величины, поскольку заряд разделяется только в этих изначально нейтральных устройствах. Конечным результатом является то, что комбинация напоминает одиночный конденсатор с эффективным разделением пластин больше, чем у отдельных конденсаторов.(См. Рисунок 1 (b).) Чем больше расстояние между пластинами, тем меньше емкость. Общей особенностью последовательного соединения конденсаторов является то, что общая емкость меньше любой из отдельных емкостей.

Рисунок 1. (a) Конденсаторы, подключенные последовательно. Величина заряда на каждой пластине — Q . (b) Эквивалентный конденсатор имеет большее расстояние между пластинами d . При последовательном соединении общая емкость меньше, чем у любого из отдельных конденсаторов.

Мы можем найти выражение для общей емкости, рассматривая напряжение на отдельных конденсаторах, показанных на рисунке 1. Решение [latex] \ boldsymbol {C = \ frac {Q} {V}} [/ latex] для [latex] \ boldsymbol {V} [/ latex] дает [латекс] \ boldsymbol {V = \ frac {Q} {C}} [/ latex]. Таким образом, напряжения на отдельных конденсаторах составляют [латекс] \ boldsymbol {V_1 = \ frac {Q} {C_1}} [/ latex], [латекс] \ boldsymbol {V_2 = \ frac {Q} {C_2}} [/ латекс ] и [латекс] \ boldsymbol {V_3 = \ frac {Q} {C_3}} [/ latex]. Общее напряжение складывается из отдельных напряжений:

[латекс] \ boldsymbol {V = V_1 + V_2 + V_3}.[/ латекс]

Теперь, называя общую емкость [латекс] \ boldsymbol {C_S} [/ latex] для последовательной емкости, примите во внимание, что

[латекс] \ boldsymbol {V =} [/ latex] [латекс] \ boldsymbol {\ frac {Q} {C_S}} [/ latex] [латекс] \ boldsymbol {= V_1 + V_2 + V_3}. [/ Латекс ]

Вводя выражения для [latex] \ boldsymbol {V_1} [/ latex], [latex] \ boldsymbol {V_2} [/ latex] и [latex] \ boldsymbol {V_3} [/ latex], получаем

[латекс] \ boldsymbol {\ frac {Q} {C_S} = \ frac {Q} {C_1} + \ frac {Q} {C_2} + \ frac {Q} {C_3}}.[/ латекс]

Удаляя [латекс] \ boldsymbol {Q} [/ latex] s, мы получаем уравнение для полной емкости в серии [латекс] \ boldsymbol {C_S} [/ latex] равным

[латекс] \ boldsymbol {\ frac {1} {C_S}} [/ latex] [латекс] \ boldsymbol {=} [/ latex] [латекс] \ boldsymbol {\ frac {1} {C_1}} [/ латекс ] [латекс] \ boldsymbol {+} [/ латекс] [латекс] \ boldsymbol {\ frac {1} {C_2}} [/ latex] [латекс] \ boldsymbol {+} [/ латекс] [латекс] \ boldsymbol { \ frac {1} {C_3}} [/ latex] [latex] \ boldsymbol {+ \ cdots}, [/ latex]

, где «…» означает, что выражение действительно для любого количества конденсаторов, подключенных последовательно.Выражение этой формы всегда приводит к общей емкости [латекс] \ boldsymbol {C_S} [/ latex], которая меньше любой из отдельных емкостей [латекс] \ boldsymbol {C_1} [/ latex], [латекс] \ boldsymbol {C_2} [/ latex],…, как показано в следующем примере.

Общая емкость в серии,

C с

Общая емкость в серии: [латекс] \ boldsymbol {\ frac {1} {C_S} = \ frac {1} {C_1} + \ frac {1} {C_2} + \ frac {1} {C_3} + \ cdots } [/ latex]

Пример 1: Что такое последовательная емкость?

Найдите общую емкость для трех последовательно соединенных конденсаторов, учитывая, что их отдельные емкости равны 1.000, 5.000 и 8.000 [латекс] \ mu \ textbf {F} [/ latex].

Стратегия

Имея данную информацию, общую емкость можно найти, используя уравнение для емкости в серии.

Решение

Ввод заданных емкостей в выражение для [latex] \ boldsymbol {\ frac {1} {C_S}} [/ latex] дает [latex] \ boldsymbol {\ frac {1} {C_S} = \ frac {1} { C_1} + \ frac {1} {C_2} + \ frac {1} {C_3}} [/ латекс].

[латекс] \ boldsymbol {\ frac {1} {C_S}} [/ latex] [латекс] \ boldsymbol {=} [/ latex] [латекс] \ boldsymbol {\ frac {1} {1.000 \; \ mu \ textbf {F}}} [/ latex] [latex] \ boldsymbol {+} [/ latex] [latex] \ boldsymbol {\ frac {1} {5.000 \; \ textbf {F}}} [/ latex] [латекс] \ boldsymbol {+} [/ latex] [латекс] \ boldsymbol {\ frac {1} {8.000 \; \ mu \ textbf {F}}} [/ latex] [латекс] \ boldsymbol { =} [/ latex] [латекс] \ boldsymbol {\ frac {1.325} {\ mu \ textbf {F}}} [/ latex]

Инвертирование для поиска [latex] \ boldsymbol {C_S} [/ latex] дает [latex] \ boldsymbol {C_S = \ frac {\ mu \ textbf {F}} {1.325} = 0.755 \; \ mu \ textbf {F} }[/латекс].

Обсуждение

Общая последовательная емкость [латекс] \ boldsymbol {C_s} [/ latex] меньше наименьшей индивидуальной емкости, как было обещано.При последовательном соединении конденсаторов сумма меньше деталей. На самом деле это меньше, чем у любого человека. Обратите внимание, что иногда возможно и более удобно решить уравнение, подобное приведенному выше, путем нахождения наименьшего общего знаменателя, который в данном случае (показаны только целочисленные вычисления) равен 40. Таким образом,

[латекс] \ boldsymbol {\ frac {1} {C_S}} [/ latex] [латекс] \ boldsymbol {=} [/ latex] [латекс] \ boldsymbol {\ frac {40} {40 \; \ mu \ textbf {F}}} [/ latex] [латекс] \ boldsymbol {+} [/ latex] [latex] \ boldsymbol {\ frac {8} {40 \; \ mu \ textbf {F}}} [/ latex] [латекс] \ boldsymbol {+} [/ латекс] [латекс] \ boldsymbol {\ frac {5} {40 \; \ mu \ textbf {F}}} [/ latex] [латекс] \ boldsymbol {=} [/ латекс] [латекс] \ boldsymbol {\ frac {53} {40 \; \ mu \ textbf {F}}}, [/ латекс]

, так что

[латекс] \ boldsymbol {C_S =} [/ latex] [латекс] \ boldsymbol {\ frac {40 \; \ mu \ textbf {F}} {53}} [/ latex] [латекс] \ boldsymbol {= 0 .755 \; \ mu \ textbf {F}}. [/ latex]

На рис. 2 (а) показано параллельное соединение трех конденсаторов с приложенным напряжением. Здесь общую емкость найти легче, чем в последовательном случае. Чтобы найти эквивалентную общую емкость [латекс] \ boldsymbol {\ textbf {C} _ {\ textbf {p}}} [/ latex], сначала отметим, что напряжение на каждом конденсаторе составляет [латекс] \ boldsymbol {V} [ / latex], то же самое, что и у источника, так как они подключаются к нему напрямую через проводник. (Проводники являются эквипотенциальными, поэтому напряжение на конденсаторах такое же, как и на источнике напряжения.Таким образом, конденсаторы имеют такой же заряд, как и при индивидуальном подключении к источнику напряжения. Общий заряд [латекс] \ boldsymbol {Q} [/ latex] равен сумме индивидуальных сборов:

[латекс] \ boldsymbol {Q = Q_1 + Q_2 + Q_3}. [/ Latex]

Рис. 2. (a) Конденсаторы, включенные параллельно. Каждый из них подключен непосредственно к источнику напряжения, как если бы он был полностью один, поэтому общая параллельная емкость — это просто сумма отдельных емкостей. (b) Эквивалентный конденсатор имеет большую площадь пластины и поэтому может удерживать больше заряда, чем отдельные конденсаторы.

Используя соотношение [латекс] \ boldsymbol {Q = CV} [/ latex], мы видим, что общий заряд составляет [латекс] \ boldsymbol {Q = C _ {\ textbf {p}} V} [/ latex], и индивидуальные расходы: [латекс] \ boldsymbol {Q_1 = C_1 V} [/ latex] , [латекс] \ boldsymbol {Q_2 = C_2 V} [/ latex] , и [латекс] \ boldsymbol {Q_3 = C_3 V} [/ латекс]. Ввод их в предыдущее уравнение дает

[латекс] \ boldsymbol {C _ {\ textbf {p}} V = C_1 V + C_2 V + C_3 V}. [/ Latex]

Исключая [латекс] \ boldsymbol {V} [/ latex] из уравнения, мы получаем уравнение для полной емкости в параллельном [латексе] \ boldsymbol {C _ {\ textbf {p}}} [/ latex]:

[латекс] \ boldsymbol {C _ {\ textbf {p}} = C_1 + C_2 + C_3 \ cdots} [/ latex].

Общая параллельная емкость — это просто сумма отдельных емкостей. (И снова «» указывает на то, что выражение действительно для любого количества конденсаторов, подключенных параллельно.) Так, например, если конденсаторы в приведенном выше примере были подключены параллельно, их емкость была бы

.

[латекс] \ boldsymbol {C _ {\ textbf {p}} = 1.000 \; \ mu \ textbf {F} + 5.000 \; \ mu \ textbf {F} + 8.000 \; \ mu \ textbf {F} = 14.000 \; \ mu \ textbf {F}}. [/ latex]

Эквивалентный конденсатор для параллельного соединения имеет значительно большую площадь пластины и, следовательно, большую емкость, как показано на Рисунке 2 (b).

Общая емкость параллельно,

C p [латекс] \ boldsymbol {C _ {\ textbf {p}}} [/ latex]

Общая емкость параллельно [латекс] \ boldsymbol {C _ {\ textbf {p}} = C_1 + C_2 + C_3 + \ cdots} [/ latex]

Более сложные соединения конденсаторов иногда могут быть последовательными и параллельными. (См. Рис. 3.) Чтобы найти общую емкость таких комбинаций, мы идентифицируем последовательные и параллельные части, вычисляем их емкости, а затем находим общую.

Рисунок 3. (a) Эта схема содержит как последовательное, так и параллельное соединение конденсаторов. См. Пример 2 для расчета общей емкости цепи. (b) C 1 и C 2 идут последовательно; их эквивалентная емкость C S меньше, чем у любого из них. (c) Обратите внимание, что C S находится параллельно с C 3 .Таким образом, общая емкость равна сумме C S и C 3 .

Смесь последовательной и параллельной емкости

Найдите общую емкость комбинации конденсаторов, показанной на рисунке 3. Предположим, что емкости на рисунке 3 известны с точностью до трех десятичных знаков ([латекс] \ boldsymbol {C_1 = 1.000 \; \ mu \ textbf {F}} [/ latex ], [латекс] \ boldsymbol {C_2 = 5.000 \; \ mu \ textbf {F}} [/ latex] и [латекс] \ boldsymbol {C_3 = 8.000 \; \ mu \ textbf {F}} [/ latex]) и округлите ответ до трех десятичных знаков.

Стратегия

Чтобы найти общую емкость, мы сначала определяем, какие конденсаторы включены последовательно, а какие — параллельно. Конденсаторы [латекс] \ boldsymbol {C_1} [/ latex] и [латекс] \ boldsymbol {C_2} [/ latex] включены последовательно. Их комбинация, обозначенная на рисунке [латекс] \ boldsymbol {C_S} [/ latex], параллельна [латексу] \ boldsymbol {C_3} [/ latex].

Решение

Поскольку [латекс] \ boldsymbol {C_1} [/ latex] и [latex] \ boldsymbol {C_2} [/ latex] соединены последовательно, их общая емкость определяется выражением [латекс] \ boldsymbol {\ frac {1} {C_S } = \ frac {1} {C_1} + \ frac {1} {C_2} + \ frac {1} {C_3}} [/ latex].Ввод их значений в уравнение дает

[латекс] \ boldsymbol {\ frac {1} {C_1}} [/ latex] [латекс] \ boldsymbol {+} [/ latex] [латекс] \ boldsymbol {\ frac {1} {C_2}} [/ латекс ] [латекс] \ boldsymbol {=} [/ латекс] [латекс] \ boldsymbol {\ frac {1} {1.000 \; \ mu \ textbf {F}}} [/ латекс] [латекс] \ boldsymbol {+} [ / latex] [латекс] \ boldsymbol {\ frac {1} {5.000 \; \ mu \ textbf {F}}} [/ latex] [латекс] \ boldsymbol {=} [/ latex] [латекс] \ boldsymbol {\ гидроразрыв {1.200} {\ mu \ textbf {F}}}. [/ latex]

Инвертирование дает

[латекс] \ boldsymbol {C _ {\ textbf {S}} = 0.833 \; \ mu \ textbf {F}}. [/ Latex]

Эта эквивалентная последовательная емкость подключена параллельно третьему конденсатору; Таким образом, общая сумма составляет

[латекс] \ begin {array} {r @ {{} = {}} l} \ boldsymbol {C _ {\ textbf {tot}}} & \ boldsymbol {C_S + C_S} \\ [1em] & \ boldsymbol { 0.833 \; \ mu \ textbf {F} + 8.000 \; \ mu \ textbf {F}} \\ [1em] & \ boldsymbol {8.833 \; \ mu \ textbf {F}}. \ end {array} [/ latex]

Обсуждение

Этот метод анализа комбинаций конденсаторов по частям, пока не будет получена общая сумма, может быть применен к более крупным комбинациям конденсаторов.

  • Общая емкость последовательно [латекс] \ boldsymbol {\ frac {1} {C _ {\ textbf {S}}} = \ frac {1} {C_1} + \ frac {1} {C_2} + \ frac {1 } {C_3} + \ cdots} [/ латекс]
  • Общая емкость параллельно [латекс] \ boldsymbol {C _ {\ textbf {p}} = C_1 + C_2 + C_3 + \ cdots} [/ latex]
  • Если схема содержит комбинацию конденсаторов, включенных последовательно и параллельно, определите последовательную и параллельную части, вычислите их емкости, а затем найдите общую сумму.

Концептуальные вопросы

1: Если вы хотите хранить большое количество энергии в конденсаторной батарее, подключите ли вы конденсаторы последовательно или параллельно? Объяснять.

Задачи и упражнения

1: Найдите общую емкость комбинации конденсаторов на рисунке 4.

Рисунок 4. Комбинация последовательного и параллельного подключения конденсаторов.

2: Предположим, вам нужна конденсаторная батарея с общей емкостью 0,750 Ф и у вас есть множество конденсаторов 1,50 мФ. Какое наименьшее число вы могли бы связать вместе, чтобы достичь своей цели, и как бы вы их связали?

3: Какую общую емкость можно получить, подключив символ [латекс] \ bold {5.00 \; \ mu \ textbf {F}} [/ latex] и конденсатор [latex] \ boldsymbol {8.00 \; \ mu \ textbf {F}} [/ latex] вместе?

4: Найдите общую емкость комбинации конденсаторов, показанной на рисунке 5.

Рисунок 5. Комбинация последовательного и параллельного подключения конденсаторов.

5: Найдите общую емкость комбинации конденсаторов, показанной на рисунке 6.

Рисунок 6. Комбинация последовательного и параллельного подключения конденсаторов.

6: Необоснованные результаты

(a) Конденсатор [латекс] \ boldsymbol {8.00 \; \ mu \ textbf {F}} [/ latex] подключен параллельно другому конденсатору, что дает общую емкость [латекс] \ boldsymbol {5.00 \; \ mu \ textbf {F}} [/ латекс]. Какая емкость у второго конденсатора? б) Что неразумного в этом результате? (c) Какие предположения необоснованны или непоследовательны?

Решения

Задачи и упражнения

1: [латекс] \ boldsymbol {0.293 \; \ mu \ textbf {F}} [/ латекс]

3: [латекс] \ boldsymbol {3.08 \; \ mu \ textbf {F}} [/ latex] в последовательной комбинации, [латекс] \ boldsymbol {13.0 \; \ mu \ textbf {F}} [/ латекс ] в параллельной комбинации

4: [латекс] \ boldsymbol {2.79 \; \ mu \ textbf {F}} [/ латекс]

6: (a) [латекс] \ boldsymbol {-3.00 \; \ mu \ textbf {F}} [/ latex]

(b) У вас не может быть отрицательного значения емкости.

(c) Предположение, что конденсаторы были подключены параллельно, а не последовательно, было неверным.Параллельное соединение всегда дает большую емкость, в то время как здесь предполагалась меньшая емкость. Это могло произойти, только если конденсаторы подключены последовательно.

Конденсаторы комбинированные

В некоторых цепях конденсаторы соединены последовательно и параллельно. Чтобы определить, например, общий заряд, накопленный набором конденсаторов, мы должны найти единственную эквивалентную емкость набора.Это делается путем идентификации пары конденсаторов в наборе, которые включены последовательно или параллельно друг другу, замены этой пары на эквивалентный конденсатор (тем самым уменьшая количество конденсаторов на один) и повторения до тех пор, пока у нас не останется один. конденсатор, который является эквивалентом набора.

Возьмем, к примеру, ситуацию, описанную выше. Четыре конденсатора имеют следующие значения:

C 1 = C 2 = 90 пФ.
C 3 = 45 пФ
C 4 = 120 пФ

Какова разность потенциалов на каждом конденсаторе? Сколько заряда у каждого конденсатора?

Чтобы решить эту проблему, нам нужно найти эквивалентную емкость набора конденсаторов.Первый шаг — перерисовать схему так, чтобы C 1 был нарисован вертикально — это делает более очевидным, что идет параллельно или последовательно.

Теперь сократите цепь с 4 конденсаторов до 1.

Шаг 1 — C 2 и C 3 идут последовательно. Замените эту пару одним конденсатором C 23 :

1 / C 23 = 1 / C 2 + 1 / C 3 = 1/90 + 1/45 = 3/90.

Следовательно, C 23 = 90/3 = 30 пФ.

Шаг 2 — C 1 и C 23 идут параллельно. Замените эту пару одним конденсатором C 123 = 90 + 30 = 120 пФ.

Шаг 3 — C 4 и C 123 идут последовательно. Замените эту пару одним конденсатором C eq :

1 / C экв = 1 / C 4 + 1 / C 123 = 1/120 + 1/120 = 2/120

C экв = 120/2 = 60 пФ

Шаг 4 — Определите заряд на C eq .

Q = C экв ΔV = 60 пФ * 12 В = 720 пКл.

Теперь нам нужно расширить схему до четырех исходных конденсаторов и определить заряд и разность потенциалов на каждом из них по мере продвижения.

Шаг 1 — C eq представляет C 4 и C 123 последовательно. Последовательные конденсаторы имеют одинаковый заряд, но разделяют разность потенциалов.

Q 4 = Q 123 = 720 пКл.

Конденсаторы одинаковые, поэтому на каждый из них подается по 6 вольт.

Шаг 2 — C 123 представляет собой C 1 и C 23 параллельно. Параллельно подключенные устройства имеют одинаковую разность потенциалов (в данном случае 6 В).

Q 1 = C 1 * 6 = 540 пКл.
Q 23 = C 23 * 6 = 180 пКл.

Они добавляют к 720 пКл, как и должно быть.

Шаг 3 — C 23 представляет последовательно C 2 и C 3 .

Q 2 = Q 3 = 180 пКл.

ΔV 2 = Q 2 / C 2 = 180/90 = 2 В.
ΔV 3 = Q 3 / C 3 = 180/45 = 4 В.

Они добавляют к 6 вольт, как должны.

Шаг 4. Хороший способ проверить согласованность — пометить потенциал в разных точках. Выберите некоторую точку в качестве ориентира (скажем, 0 В на отрицательном полюсе батареи) и пометьте другие точки, относящиеся к ней.Убедитесь, что разность потенциалов на конденсаторах соответствует этим значениям потенциала.

Конденсаторы — несколько конденсаторов

< >

Как следует подключить нескольких конденсаторов ? Что происходит с общей емкостью в серийный номер и параллельных цепей ? Как можно увеличить всего номинальное напряжение ? Будет ли серийный или параллельный магазин больше общей энергии ?

Параллельные конденсаторы

Конденсаторы, подключенные параллельно, добавят шт. емкость вместе.

C всего = C 1 + C 2 + … + C n

параллельная цепь — самый удобный способ увеличить общее хранилище электрический заряд.

Общее напряжение рейтинг не меняется. Каждый конденсатор будет «видеть» одно и то же напряжение. Они все должно быть рассчитано, по крайней мере, на напряжение вашего источника питания. И наоборот, вы не должны прикладывать больше напряжения, чем наименьшее значение напряжения среди параллельных конденсаторов.

Конденсаторы серии

Конденсаторы, соединенные последовательно, будут иметь нижний общая емкость, чем любая отдельная в цепи.

Если у вас есть только два конденсатора последовательно, это уравнение можно упростить до:

Если у вас последовательно установлены два одинаковых конденсатора , это дополнительно упрощается до:

Эта последовательная схема предлагает более высокий общий уровень напряжения.Падение напряжения на каждом конденсаторе складывается из общее приложенное напряжение.

Осторожно: Если конденсаторы разные, напряжение разделится так, что конденсаторы меньшего размера подбросьте больше напряжения! Это потому, что все они получают тот же зарядный ток, а напряжение обратно пропорционально пропорционально емкости.

Еще хуже , если один конденсатор немного негерметичен, он будет постепенно передавать свое напряжение другим, возможно превышение их номинального напряжения в свою очередь.И если один из них пробивает диэлектрический барьер и может повредить другие каскадно. Вот почему конденсаторы серии обычно не используются в силовых цепях.

Сеть резисторов для конденсаторов серии

Но последовательная сеть просто слишком привлекательно, когда у вас ограниченные деньги и запасные части. Как ты можешь встроить какую-нибудь безопасность ?

При последовательном подключении конденсаторов любые расхождения в значениях заставляет каждый заряжаться с разной скоростью и с разным Напряжение.Разница может быть довольно большой для электролитов. Наверху из этого, как только батарея заряжена, утечка каждого конденсатора ток также вызывает * различное * напряжение на каждом конденсаторе.

Если вы полностью заряжаете банк серии, некоторые ограничения всегда недозаряженные и некоторые завышенные (не хорошо). Чтобы помочь им поделиться напряжение одинаково, вы добавляете балансировочных резисторов . В основном резисторы действуют как большой делитель напряжения и противодействуют эффектам изменения емкости и тока утечки.А если нет ток утечки, конденсаторы должны со временем стать заряженными в соответствии со значениями делителя напряжения.

Используйте это уравнение со стр.13 этого отличного руководства, предоставленного Корнелл Дубилье, «Алюминий Руководство по применению электролитического конденсатора »для расчета балансировочные резисторы:

Для 2 конденсаторов последовательно: R = (2V m — V b ) / (0.0015 C V b )
Для конденсаторов N> 2 : R = (NV м — V b ) / (0,0015 C V b )
где R = сопротивление в МОм
В м = максимальное напряжение, которое вы разрешаете на любом конденсаторе
В b = максимальное напряжение на всей банке из двух (или N) конденсаторов
N = количество конденсаторов в серии
C = емкость в мкФ

Пример: Предположим, у вас есть два одинаковых 1000 мкФ конденсаторы, и соедините их последовательно, чтобы удвоить напряжение номинал и уменьшить вдвое общую емкость.Предположим также, что они рассчитаны на 100 Вт постоянного тока (рабочее напряжение) и максимальное перенапряжение 125 В. Решите уравнение, используя V м = 125 и V b = 200.

Решение: R = (2×125 — 200) / (0,0015 x 1000 x 200) = 50/300 = 0,167 M = 167 кОм

Некоторые связанные последствия в этом примере:

  • Резисторы в этом примере будут давать нагрузку I = 200В / (2 * 167К) = 0,6 мА на системе зарядки.Здесь нет проблем, скорее всего, это незначительно к мощной системе зарядки койлгана.
  • Балансировочные резисторы потекут по крышке. заряд с постоянной времени RC 2мин 47сек, что означает Вы должны держать зарядное устройство подключенным до момента выстрела. Они действуют как предохранительный резистор для удаления воздуха, который гарантирует, что колпачки не остаются заряженными на следующий день (или неделю, или месяц!). Кроме того, это предотвращает зарядку конденсаторов до несколько вольт из-за эффекта диэлектрической памяти.
  • Каждый резистор выделяет тепло ( P = I 2 R ) в этом примере номинал 60 мВт, худший случай 120 мВт. Так что вы следует использовать резистор на 1 Вт или больше. Ладно, резистор на 1/2 ватта тоже подойдет, но может сильно нагреться.
  • Термин «0,0015 C V b » означает оценка (в микроамперах) разницы токов утечки в двух конденсаторах, включенных последовательно при номинальной температуре.
  • Неисправный конденсатор с током утечки более примерно 1 мА собирается перезарядить другую шапку.Так что будь осторожен, и периодически проверяйте актуальные напряжения!
  • Плохой конденсатор с внутренним коротким замыканием перезаряжается другие конденсаторы (если он не открывается) независимо от того, что резисторы, которые вы используете.

Общая энергия серии по сравнению с параллельной

Давайте посмотрим, может ли последовательная или параллельная цепь хранить больше полная энергия.

Напомним, что энергия в одном конденсаторе пропорциональна квадрат напряжения.Заманчиво использовать последовательные конденсаторы для получить прирост энергии, используя «квадрат напряжения» в наших интересах. Но давайте посмотрим внимательнее …

Предположим, у вас есть два одинаковых конденсатора емкостью C и номинальное напряжение В . (Конденсаторы не должны быть идентичны, но результаты верны для общего случая, и математика таким образом намного проще.) Давайте посчитаем запасенную энергию E для обеих схем.

  1. Накопленная энергия в двух параллельных конденсаторах, заряженных до напряжения В:
  2. Накопленная энергия в двух последовательных конденсаторах, заряженных до напряжения 2 В:

Есть нет разницы ! Обе схемы хранят одно и то же количество энергии. Это должно подтвердить здравый смысл, это говорит о том, что вы не можете увеличить общий запас энергии только за счет повторное подключение одних и тех же конденсаторов в разных схемах.

Выводы

Параллельные конденсаторы безопаснее безопаснее и надежнее последовательного соединения.

Нет преимущества в общем накоплении энергии, чтобы выбрать один из этих цепей над другой. Но! Вполне может быть время когда вам нужна более низкая емкость (например, более быстрый синхронизирующий импульс) и более высокое напряжение, чем может обеспечить имеющаяся у вас деталь.

конденсаторов последовательно и параллельно с примерами

Последовательные и параллельные конденсаторы

Системы с более чем одним конденсатором имеют эквивалентную емкость.Конденсаторы можно соединять между собой двумя способами. Их можно подключать последовательно и параллельно. Сначала мы увидим конденсаторы, включенные параллельно.

В этой цепи конденсаторы включены параллельно.


Потому что левые стороны конденсаторов подключены к потенциалу a, а правые стороны конденсаторов подключены к потенциалу b.Другими словами, мы можем сказать, что каждый конденсатор имеет одинаковую разность потенциалов. Находим заряд каждого конденсатора как;

Q 1 = C 1 .V

Q 2 = C 2 .V

Q 3 = C 3 .V

Общий заряд системы определяется сложением каждого заряда.

Q всего = C экв .V

Q итого = Q 1 + Q 2 + Q 3 = C 1 .V + C 2 .V + C 3 .V = V. (C 1 + C 2 + C 3 ) = C экв.

C экв = C 1 + C 2 + C 3

Как видите, мы нашли эквивалентную емкость системы как C 1 + C 2 + C 3

Теперь посмотрим последовательно включенные конденсаторы;

В конденсаторах, подключенных последовательно, каждый конденсатор имеет одинаковый поток заряда от батареи.В этой схеме заряд + Q течет от положительной части батареи к левой пластине первого конденсатора и притягивает заряд –Q на правой пластине, с той же идеей заряд -Q течет от батареи к правой пластине. третьего конденсатора и притягивает + Q на левой пластине. Таким же образом заряжаются и другие конденсаторы. Подводя итог, можно сказать, что каждый конденсатор имеет одинаковый заряд с батареей.

C 1 .V 1 = Q

С 2 .V 2 = Q, V = V 1 + V 2 + V 3 и Q = C экв .V

C 3 .V 3 = Q


Пример: вычислить эквивалентную емкость между точками a и b.

Пример: В приведенной ниже схеме C1 = 60 мкФ, C2 = 20 мкФ, C3 = 9 мкФ и C4 = 12 мкФ.Если разность потенциалов между точками a an b Vab = 120V, найдите заряд второго конденсатора.

Электростатические исследования и решения

Емкость и конденсаторы <Пред. Далее> Памятка по электростатике

Физика для науки и техники II

Пример из отдела академических технологий на Vimeo.

Пример подключения конденсаторов

Давайте рассмотрим пример, связанный с подключением конденсаторов. Предположим, что у нас есть цепь с источником питания, который генерирует v вольт разности потенциалов, подключенной к конденсатору c 1. Скажем, таким образом c 2, c 3, c 4 и c5. Допустим, у нас есть еще один конденсатор с емкостью c 6. Мы хотели бы вычислить эквивалентную емкость этой схемы.

Приведем числовые значения этих конденсаторов.Скажем, c 1 равно 1 мкФ, c 2 равно 2 мкФ, а c 3 равно 2 мкФ. c 4 равно 3 мкФ, а c 5 равно 5 мкФ. Наконец, c 6 равно 1 мкФ. Когда мы смотрим на нашу схему, мы сразу понимаем, что c 3, c 4 и c 5, эти три, соединены параллельно. Таким образом, эквивалент этой параллельной комбинации подключается последовательно к c 1, c 2 и c6. Решая эти типы проблем или анализируя эти типы схем, мы можем продолжить и вычислить эквивалентную емкость соединений конденсаторов внутри схемы, и на каждом этапе мы перерисовываем схему.

Сначала давайте посчитаем эквивалентную емкость этой параллельной комбинации. Другими словами, просто выньте этот блок и замените его эквивалентной емкостью. По мере того, как мы это делаем, давайте перерисуем схему. Тогда наша схема будет равна c 1 здесь, c 2 здесь, а вот эквивалент этой параллельной комбинации, и давайте назовем эту схему c эквивалентом 1 и продолжим, c 6 здесь. Все они теперь соединены последовательно. Поскольку эти три c 3, c 4 и c 5 соединены параллельно, мы можем мгновенно вычислить их эквивалентную емкость для параллельного подключения, эквивалент c — это непосредственно сумма емкостей каждого конденсатора, поэтому эквивалент c равен c 3 плюс c 4 плюс c 5.Следовательно, эквивалент C 1 будет равен c 3 равен 2 микрофарадам, плюс c 4 равен 3 микрофарадам, плюс c 5 равен 5 микрофарадам, поэтому эквивалент c будет равен 10 микрофарадам.

После того, как мы определим c эквивалент 1, теперь мы можем легко продолжить и вычислить эквивалентную емкость этих четырех конденсаторов, которые соединены последовательно. Мы знаем, что для последовательного соединения величина, обратная эквивалентной емкости, равна сумме обратных величин каждой емкости в комбинации.Поэтому мы заменим все эти четыре на одну, и это будет полная эквивалентная емкость этой схемы. Наша последняя упрощенная схема примет эту форму.

1 по сравнению с эквивалентом c, следовательно, будет равно 1 по сравнению с эквивалентом c 1 плюс 1 по сравнению с c 2 плюс 1 по сравнению с эквивалентом c 1 плюс 1 вместо c 6. Если мы подставим числовые значения, 1 вместо эквивалента c будет равно c 1 было равно 1 микрофараду, а c 2 равно 2 микрофарадам, поэтому у нас 1 больше 1 плюс 1 больше 2 плюс 1 больше c эквивалентно 1 и это 10 микрофарад, 1 больше 10, плюс c 6 и c 6 были равны 1 микрофараду, поэтому 1 больше 1.

Добавить комментарий

Ваш адрес email не будет опубликован. Обязательные поля помечены *